Sunteți pe pagina 1din 56

englishforeveryone.

org Name________________
Date________________


Beginning Critical Reading – Paper
Paper has many uses. People draw on paper. People paint on paper. People write on
paper. People clean with paper. People make boxes out of paper. People blow their noses on
paper. Books are made of paper. Magazines are made of paper. Some money is printed on paper.
Sandpaper is made of paper. Wallpaper is made of paper.
5 Some paper is made from plants. Some paper is made from cotton. Some paper is made
from bamboo. Some paper is made from wheat. Most paper is made from trees. 95% of paper is
made from trees. Some paper is made from pine trees. Some paper is made from oak trees. Some
paper is made from maple trees. Some paper is recycled. When paper is recycled, new paper is
made from old paper.
10 Some people say they work in paper-free offices. Some people think computers will take
away the need for paper. But people will always need to blow their noses. People will want to
sand surfaces with sandpaper. People will want to paint or draw on paper. People will want to
put things in paper boxes. So, people will probably need to use paper for many more years.

Questions
1. According to the passage, which of the following statements is/are true?
I) Ninety-five percent of paper is made from pine trees.
II) Computers will take away the need for paper.
III) There will probably always be uses for paper.

A) I only
B) II only
C) III only
D) I and II only
E) II and III only

2. What is the main idea of the second paragraph?


A) What is made of paper
B) What paper is made of
C) What people think of paper
D) The many uses of paper
E) The future of paper

3. In line 12, sand most closely means

A) grainy mineral
B) paint or color
C) polish or smooth
D) use for storage
E) draw
Answers and Explanations

1. The correct answer is C.


I) Incorrect. According to line 7, “Ninety–five percent of paper is made from trees.
Some paper is made from pine trees.”
II) Incorrect. According to lines 10-11, “Some people think computers will take away
the need for paper.” So while it might be true that computers will take away the
need for paper, the passage states that it is some people’s opinion, not a fact.
III) Correct. According to lines 13-14, “People will probably always need to use paper.”

A) I only
B) II only
C) III only
D) I and II only
E) II and III only

2. The correct answer is B.


A) Incorrect. The second paragraph tells what paper is made of. For example, according to
line 5, “Some paper is made from plants. Some paper is made from cotton.” The first
paragraph tells what is made of paper. According to lines 3-4, “Books are made of paper.
Magazines are made of paper.”
B) Correct. The second paragraph tells what paper is made of. For example, according to
line 5, “Some paper is made from plants. Some paper is made from cotton.”
C) Incorrect. The second paragraph tells what paper is made of. For example, according to
line 5, “Some paper is made from plants. Some paper is made from cotton.” The third
paragraph contains some information about what people think. For example, according
to lines 10-11, “Some people think computers will take away the need for paper.”
D) Incorrect. The second paragraph tells what paper is made of. For example, according to
line 5, “Some paper is made from plants. Some paper is made from cotton.” The first
paragraph tells about the many uses of paper. For example, according to line 1, “Paper
has many uses. People draw on paper. People paint on paper.”
E) Incorrect. The second paragraph tells what paper is made of. For example, according to
line 5, “Some paper is made from plants. Some paper is made from cotton.” The third
paragraph discusses the future of paper. For example, according to lines 10-11, “Some
people think computers will take away the need for paper. But people will always need to
blow their noses.”

3. The correct answer is C.

A) Incorrect. In lines 11–12, the word sand is used as a verb. According to the passage,
“Some people will want to sand surfaces with sandpaper.” While the word sand does
sometimes refer to a mineral, that usage of sand is a noun, not a verb. So in this case,
the word sand cannot mean a grainy mineral.
B) Incorrect. While it is true that people paint or color on surfaces and that there is a
reddish–brown color which is called sand, paint or color is probably not the meaning of
sand as it is used in this line, because the following sentence states that “Some people
will want to paint or draw on paper.”
C) Correct. The meaning of sand in this sentence is to smooth or polish. Since sandpaper
has a rainy surface, it smoothes or polishes other surfaces when it is rubbed against them.
D) Incorrect. Since the third paragraph contains a listing of possible future uses of paper,
storage does appear as one of the possible uses. However, it appears in line 13 – “Some
people will want to put things in paper boxes.” Since putting things in boxes is storing
things, this is the sentence where storage is listed. So it is unlikely that sand means use
for storage.
E) Incorrect. Since the third paragraph is a listing of possible future uses of paper,
drawing on paper does appear as one of the possible future uses. However it is used in
lines 13-14, “Some people will want to paint or draw on paper.” So sand probably does
not refer to drawing.
englishforeveryone.org Name________________
Date________________

Beginning Critical Reading – The Sun
The sun is a star. The sun is at the center of the solar system. The sun is the largest object
in the solar system. It is more than 99.8% of the mass of the solar system. More than one million
earths could fit inside the sun!
From earth, the sun looks like a yellow ball in the sky. A long time ago, people didn’t
5 know what the sun was. Many people told stories about the sun. People in many countries told
stories about the sun. In some stories, people said the sun was a god. Some people gave names to
the sun. The Greeks named it Helios. The Romans named it Sol. The name Solar System comes
from the Roman name Sol.
The sun is very hot. On the surface, it is about 5,510 ° Celsius. That’s equal to about
10 11,000 ° Fahrenheit. The inside of the sun is even hotter. The core of the sun is 15,000,000 °C!
That’s 27,000,000 °F!
The light from the sun is very bright. People must not look directly at the sun. Looking
directly at the sun will hurt your eyes.
People need the sun’s heat and light to live. Animals need the sun’s heat and light to live.
15 Plants need the sun’s heat and light to live. Plants make food with sunlight. People and animals
eat the plants. Plants also use the sun to make oxygen. People and animals need to breathe
oxygen.
Today people do not tell stories about the sun. Today people do not think the sun is a god.
But, people know that the sun is necessary for life on earth.
20
Questions
1. According to the passage, which of the following statements is/are true?
I) The sun is over a million times brighter than the earth.
II) Some people used to think the sun was a god.
III) The sun’s surface is cooler than its core.
A) I only
B) II only
C) III only
D) I and II only
E) II and III only
2. The main idea of the second paragraph is to explain
A) what the sun is.
B) what the sun looks like.
C) how people reacted to the sun.
D) who the Greeks and Romans were.
E) who did not know what the sun was.
3. In line 10, core most closely means
A) center
B) gas
C) oxygen
D) surface
E) temperature
Answers and Explanations

1. The correct answer is E.


I) Incorrect. While lines 2-3 state that more than a million earths could fit inside the
sun, there is no mention there that the sun is a million times brighter than the earth.
And while line 12 states that the light from the sun is very bright, it does not
compare the brightness of the sun to the brightness of the earth. So while the sun
may be a million times brighter than the earth, there is no such statement in the
passage.
II) Correct. Line 6 states that in some stories, people said the sun was a god.
III) Correct. Lines 9-11 state that the surface temperature is 5510 °C or 11,000 °F,
whereas the core, or inside of the sun is 15,000,000 °C or 27,000,000 °F. So the
surface of the sun is much cooler than its core.

A) I only
B) II only
C) III only
D) I and II only
E) II and III only

2. The correct answer is C.


A) Incorrect. While lines 4-5 state that a long time ago, people didn’t know what the sun
was, there is no explanation in paragraph 2 of what the sun is.
B) Incorrect. While line 4 states that from earth, the sun looks like a yellow ball in the sky,
this information is given only as background information, not as the main idea of the
paragraph.
C) Correct. The first sentence of the paragraph tells what the sun looks like. The remainder
of the paragraph tells about people’s reactions to the sun – that they didn’t know what it
was, that they told stories about it; also that some people thought it was a god, and that
some people named it.
D) Incorrect. While line 6 states “The Greeks named it Helios. The Romans named it Sol,”
there is no explanation of who the Greeks and Romans were.
E) Incorrect. While lines 4-5 state that a long time ago, people did not know what the sun
was, there is no mention of the specific people who did not know what it was. While
lines 6-7 state that the Greeks and Romans gave names to the sun, the passage does not
specify that they did not know what the sun was.

3. The correct answer is A.

A) Correct. Lines 9 and 10 compare the surface temperature of the sun with the temperature
of its core: “The inside of the sun is even hotter. The core of the sun is 15,000,000 °C!”
B) Incorrect. While the sun is made of gas, in this passage, the gas is not mentioned.
Instead, this section compares the surface of the sun to the core of the sun. Since both the
surface and the core would be made of the same material, the meaning gas would not
make sense for the word core.
C) Incorrect. Lines 9 and 10 compare the surface temperature of the sun with the
temperature of its core. Lines 16-17 state that plants use sunlight to make oxygen, not
that the sun itself is made of oxygen.
D) Incorrect. Lines 9-10 compare the temperature of the surface of the sun with the
temperature of the core. Since the comparison is made between the surface and the core,
the word core could not mean surface.
E) Incorrect. Lines 9-10 compare the heat level (which is the temperature) of the surface of
the sun with the heat level of the core of the sun. Since the comparison is being made of
the temperature of the surface and the temperature of the core, the word core could not
mean temperature.
englishforeveryone.org Name________________
Date________________


Beginning Critical Reading - Water
Water is the most common liquid in the world. Lakes contain water. Rivers contain
water. Ponds contain water. Canals contain water. Oceans contain water. The water in rivers,
ponds, and canals is fresh water. The water in oceans is salt water.
About 70% of the world is covered by water. About 97% of water in the world is salt
5 water. Only 3% of the world’s water is fresh water. Fresh water is not salty.
People usually think of water as a liquid. But water freezes to form a solid. The solid is
called ice. And water boils to form a gas. The gas is called steam or vapor.
Clouds are made of water. Water falls from clouds as rain. When it is cold, water falls as
snow. Sometimes water falls as sleet. Sleet is partly water and partly ice. Sometimes water falls
10 as hail. Hail is ice.
People need water to live. Animals need water to live. Insects need water to live. Plants
need water to live. All living things in the world need water to live.
People all over the world need to drink clean water to live. About one billion people in
the world do not have clean drinking water. The largest cause of preventable human death in the
15 world is drinking water that is unsanitary.

Questions
1. According to the passage, which of the following statements is/are true?
I) Ninety-seven percent of snow is made of salt water.
II) The largest cause of preventable human death is drinking water that is unsanitary..
III) About 1,000,000,000 people in the world do not have clean water to drink.

A) I only
B) II only
C) III only
D) I and II only
E) II and III only

2. What is the main idea of the first paragraph?


A) Fresh water is necessary for life.
B) Most water in the world is salt water.
C) Water is the most common liquid in the world.
D) People cannot drink most of the world’s water.
E) Only 3% of the world’s water is not salt water.

3. In line 15, unsanitary most closely means

A) not available
B) not clean
C) not in liquid form
D) used for another purpose
E) salty
Answers and Explanations

1. The correct answer is E.


I) Incorrect. While line 3 states that 97% of the world’s water is salty, there is no
information in the passage concerning the amount of salt, if any, in snow.
II) Correct. Lines 13-14 state that the largest cause of preventable human death is
drinking water that is unsanitary. Unsanitary means not clean.
III) Correct. Lines 12-13 state that one billion people in the world do not have clean
drinking water. One billion is the written form of the number 1,000,000,000.

A) I only
B) II only
C) III only
D) I and II only
E) II and III only

2. The correct answer is C.


A) Incorrect. While the last paragraph states that water is necessary for life, this information
is not given in the first paragraph.
B) Incorrect. While line 3 states that most of the world’s water is salt water, this is not the
main idea of the paragraph.
C) Correct. The first sentence of the paragraph states that water is the most common liquid
in the world. Lines 2-4 expand that idea. Therefore, it is the main idea of the paragraph.
D) Incorrect. While it is true that people do not drink salt water, and it is true that 97% of the
world’s water is saltwater, as stated in line 3, this information is not stated in paragraph
1.
E) Incorrect. While lines 3-4 state that only 3% of the world’s water is not salty, this
information is given as support for the main idea – that water is the most common liquid
in the world.

3. The correct answer is B.

A) Incorrect. While it is possible that water is not available to people in many places in the
world because only 3% of the world’s water is not salty, there is no mention in the
passage that water is not available.
B) Correct. Unsanitary water is not clean.
C) Incorrect. While lines 5-9 give information about water that is not in liquid form, there is
no mention that these forms of water would cause death.
D) Incorrect. Although it is true that water is used for many purposes other than drinking,
there is no mention of these purposes in the passage.
E) Incorrect. While lines 2-4 discuss salt water, and while drinking salt water may cause
death, there is no mention in the passage that salt water causes death.
englishforeveryone.org Name________________
Date________________


Intermediate Critical Reading – Hybrid Vehicles
A hybrid vehicle is a vehicle which uses two or more kinds of propulsion. Most hybrid
vehicles use a conventional gasoline engine as well as an electric motor to provide power to the
vehicle. These are usually called hybrid-electric-vehicles, or HEVs. Hybrids use two types of
propulsion in order to use gasoline more efficiently than conventional vehicles do. Most hybrid
5 vehicles use the gasoline engine as a generator which sends power to the electric motor. The
electric motor then powers the car. In conventional vehicles, the gasoline engine powers the
vehicle directly.
Since the main purpose of using a hybrid system is to efficiently use resources, most
hybrid vehicles also use other efficient systems. Most hybrid vehicles have regenerative braking
10 systems. In conventional vehicles, the gasoline engine powers the brakes, and the energy used in
braking is lost. In regenerative braking systems, the energy lost in braking is sent back into the
electrical battery for use in powering the vehicle. Some hybrid vehicles use periodic engine shut-
off as a gas–saving feature. When the vehicle is in idle, the engine temporarily turns off. When
the vehicle is put back in gear, the engine comes back on. Some hybrids use tires made of a stiff
15 material which rolls easily and prevents drag on the vehicle.
Hybrid vehicles save up to 30% of the fuel used in conventional vehicles. Since hybrid
vehicles use less gasoline, the cost of operating them is less than the cost of operating
conventional vehicles. Therefore, hybrid vehicles are gaining in popularity. According to a
recent study, over the five years it typically takes for a person to pay for a car, a typical hybrid
20 car driver would save over $6,000 in gasoline costs. Almost all the world’s major automakers are
planning and producing safe and comfortable hybrid vehicles to meet the demand for these
increasingly popular vehicles.
Although hybrid vehicles do represent a marked improvement in environmentally
conscious engineering, there still remains one significant potential drawback: battery disposal.
25 Batteries are difficult to dispose of in an environmentally safe manner. To properly dispose of
the battery in a hybrid car requires substantial effort. If the battery is not disposed of properly,
the environmental impact of a hybrid car can be equal, if not greater than, that of a regular gas
only car.
Since hybrid vehicles use less gasoline than conventional vehicles, they put fewer
30 emissions into the atmosphere than conventional vehicles do. As hybrids become more popular,
conventional vehicles are being used less, and the level of emissions being put into the air is
decreasing. Hybrid vehicles are an example of an energy-efficient technology that is good for
both consumers and the environment.
Questions
1. According to the passage, which of the following statements is/are true?
I) Two braking systems are used in most hybrid vehicles.
II) Approximately 30% of vehicles on the road are hybrid vehicles.
III) Some HEVs have engines which turn off when the vehicle is not moving.

A) I only
B) II only
C) III only
D) I and II only
E) II and III only

2. According to the passage, HEVs use two types of propulsion mainly in order to
A) go faster.
B) use gasoline efficiently.
C) provide a comfortable ride.
D) provide a safe driving experience.
E) put fewer emissions into the atmosphere.

3. In line 9, regenerative most closely means

A) electric
B) gasoline
C) powerful
D) restorative
E) second–generation

4. In the context of the passage, which of the following best articulates how the author regards
the topic?
A) Conventional vehicles may be more powerful than hybrid vehicles, but hybrid vehicles
are the more socially responsible vehicles to operate.
B) Since hybrid vehicles use less gasoline and put fewer emissions into the atmosphere than
conventional vehicles, they are better for drivers and for the environment.
C) Conventional vehicles are faster than hybrid vehicles, but hybrid vehicles are better for
the environment than conventional vehicles.
D) Since hybrid vehicles are much less expensive to purchase and operate than conventional
vehicles, they are a smarter buy than conventional vehicles.
E) Two sources of propulsion provide more power to a hybrid vehicle, making it more
powerful and faster than a conventional vehicle, so it is more socially responsible to buy
a conventional vehicle.
Answers and Explanations

1. The correct answer is C.


I) Incorrect. According to line 1, “A hybrid vehicle is a vehicle which uses two or
more kinds of propulsion,” not braking systems. While line 9 states that “Most
hybrid vehicles have regenerative braking systems,” there is no mention that they
use any additional type of braking system.
II) Incorrect. While line 16 states that “Hybrid vehicles save up to 30% of the fuel used
in conventional vehicles,” there is no mention of the percentage of hybrid vehicles
on the road.
III) Correct. Some HEVs have engines which turn off when the vehicle is not moving.
According to line 13, “When the vehicle is in idle, the engine temporarily turns
off.”

A) I only
B) II only
C) III only
D) I and II only
E) II and III only

2. The correct answer is B.


A) Incorrect. There is no mention of the vehicles’ speed anywhere in the passage.
B) Correct. According to line 8, “the main purpose of using a hybrid system is to efficiently
use resources,” and lines 16–17 and 23 state that “hybrid vehicles use less gasoline,” and
since the fuel source, or resource, of conventional vehicles is gasoline, this is the correct
answer.
C) Incorrect. While line 21 states that “automakers are planning and producing safe and
comfortable hybrid vehicles,” line 8 states that “the main purpose of using a hybrid
system is to efficiently use resources.”
D) Incorrect. While line 21 states that “automakers are planning and producing safe and
comfortable hybrid vehicles,” line 8 states that “the main purpose of using a hybrid
system is to efficiently use resources.”
E) Incorrect. While lines 29-30 state that hybrid vehicles “put fewer emissions into the
atmosphere than conventional vehicles do,” line 8 states that “the main purpose of using
a hybrid system is to efficiently use resources.”

3. The correct answer is D.

A) Incorrect. According to lines 11–12, “In regenerative braking systems, the energy lost in
braking is sent back into the electrical battery for use in powering the vehicle.” If the
braking systems were electric, the energy would come from the electric system; it would
not be sent to the electrical system. So regenerative cannot mean electric.
B) Incorrect. According to lines 10–12, “In conventional vehicles, the gasoline engine
powers the brakes, and the energy used in braking is lost. In regenerative braking
systems, the energy lost in braking is sent back into the electrical battery for use in
powering the vehicle.” Since gasoline engines are compared to systems which use
regenerative braking, gasoline cannot mean the same as regenerative.
C) Incorrect. Lines 8–9 state that “most hybrid vehicles also use other efficient systems.
Most hybrid vehicles have regenerative braking systems.” Although it is possible that an
efficient system could be powerful, power is not its main attribute – efficiency is. So it is
unlikely that regenerative means powerful.
D) Correct. According to lines 11–12, “In regenerative braking systems, the energy lost in
braking is sent back into the electrical battery for use in powering the vehicle.” So the
energy puts back or restores the electrical battery’s power, which means that the word
regenerative is likely to mean restorative.
E) Incorrect. Since the word regenerative contains the prefix re– and the root word
generative, it seems logical that it could mean second–generation, especially coupled
with the idea that hybrid vehicles are a new type or new generation of vehicles. However,
the sentence in lines 11–12, “In regenerative braking systems, the energy lost in braking
is sent back into the electrical battery for use in powering the vehicle” seems to indicate
that the word regenerative describes the action or function of the braking system rather
than its place in vehicular genealogy.

4. The correct answer is B.

A) Incorrect. While it may be true that conventional vehicles are more powerful than hybrid
vehicles, there is no mention in the passage that conventional vehicles are more
powerful than hybrid vehicles.
B) Correct. In the last paragraph, the author makes all of these points. Lines 16-17 state that
“Since hybrid vehicles use less gasoline than conventional vehicles, they put fewer
emissions into the atmosphere than conventional vehicles do.” Lines 32-33 state that
“Hybrid vehicles are an example of an energy–efficient technology that is good for both
consumers and the environment.” The author does mention the potential drawbacks in
improper battery disposal, but this does not contradict their overall view regarding the
benefit of hybrid vehicle usage.
C) Incorrect. While it may be true that conventional vehicles are faster than hybrid vehicles,
there is no mention in the passage that conventional vehicles are faster than hybrid
vehicles.
D) Incorrect. While lines 16-17 state that “Since hybrid vehicles use less gasoline, the cost of
operating them is less than the cost of operating conventional vehicles,” there is no
mention of the cost of purchasing hybrid vehicles.
E) Incorrect. There is no mention in the passage that two sources of propulsion make the
hybrid vehicle either more powerful or faster.
englishforeveryone.org Name________________
Date________________


Intermediate Critical Reading - Photography
In 1760, a man named Tiphaigne de la Roche made a bizarre prediction. In an imaginary
story called Giphantie, mirror images of scenes from nature could be captured permanently on a
canvas covered with a sticky material. After the material dried in darkness, the image would
remain on the canvas forever. At the time, the idea was unheard of. It was not until the following
5 century that the concept of photography was born, starting with some experiments by Nicéphore
Niépce.
Nicéphore Niépce, who was a French inventor, was interested in lithography, which is a
printmaking technique. He was experimenting with lithography when he found a way of copying
etchings onto glass and pewter plates using a chemical that changes when it is exposed to light.
10 He learned to burn images onto the plates and then print the images on paper. He shared his
findings with Louis Jacques Mandé Daguerre, who improved the process and announced it to the
French Academy of Sciences in 1839. The Daguerreotype, the photography method named after
Daguerre, met with great success. It was so successful, in fact, that French newspapers said the
French public had an illness called Daguerreotypomania! Daguerreotypes were inexpensive and
15 were suitable for portraiture. People called the Daguerreotype a “mirror with a memory.” Some
portrait artists went out of business when Daguerreotypes came into vogue. Others became
Daguerreotypists, now known as photographers.
Photography took off and today includes You Tube fans, I–Reporters, and everybody
with a cell phone camera. Today photography captures life around the globe. The bizarre
20 prediction of Mr. de le Roche has come true, and then some.
But how was the prediction of mirror-image pictures made in the first place? Was de la
Roche’s prediction of mirror-image photography just a lucky guess? Or was the Daguerreotype a
picture-perfect case of life imitating art?
Questions
1. According to the passage, which of the following statements is/are true?
I) Photographic images were shown in de la Roche’s book Giphantie.
II) Nicéphore Niépce experimented with printmaking.
III) Louis Jacques Mandé Daguerre invented photography.

A) I only
B) II only
C) III only
D) I and II only
E) II and III only

2. In line 1, bizarre most closely means

A) common
B) humorous
C) obvious
D) popular
E) strange

3. In the context of the passage, which of the following best articulates the author’s opinion?
A) It is hard to know how de le Roche predicted a photographic-type process a full century
before photography was invented.
B) It is hard to draw a comparison between early Daguerreotypists and modern-day
photographers.
C) It was inaccurate to say that the Daguerreotype was a “mirror with a memory,” since the
images were not reversed.
D) It is difficult to understand why the French public preferred Daguerreotypes to painted
portraiture.
E) It was difficult for portrait artists to acquire the skills necessary to become
Daguerreotypists.
Answers and Explanations

1. The correct answer is B.


I) Incorrect. Photographic images were not shown in de la Roche’s book Giphantie.
According to lines 1-3, “In an imaginary story called Giphantie, mirror images of
scenes from nature could be captured permanently on a canvas …” Since the story
was an imaginary story about images, the images were not shown. Further, according
to line 1, the story was a prediction, which tells about something that does not yet
exist.
II) Correct. According to lines 6-7, “Nicéphore Niépce, who was a French inventor,
was interested in lithography, which is a printmaking technique. He was
experimenting with lithography …” Since he was experimenting with lithography,
and lithography is a type of printmaking, then he was experimenting with
printmaking.
III) Incorrect. It is not stated in the passage that Daguerre invented photography.
According to line 10, Daguerre improved the process which had been shared with
him by Nicéphore Niépce.

A) I only
B) II only
C) III only
D) I and II only
E) II and III only

2. The correct answer is C.


A) Incorrect. The prediction could not have been common. According to line 4, “At the
time, the idea was unheard of.” When something is unheard of, it is very uncommon or
completely nonexistent.
B) Incorrect. The story was imaginary (line 1), and the idea was unheard of (line 4), and
while some imaginary stories might be humorous, there is nothing which indicates that
the prediction was humorous.
C) Incorrect. Something which is obvious is easy to see. Since the prediction was made in an
imaginary story (line 1), the idea was unheard of (line 4), and photography was not
invented until the following century (lines 4-5), all stated in the first paragraph, the
prediction could not have been obvious.
D) Incorrect. The prediction could not have been popular. In order for something to be
popular, it must be well-known. Since this idea was unheard of, according to line 4, it
was not well-known, and therefore it was not popular.
E) Correct. The prediction was strange.

3. The correct answer is A.

25 A) Correct. The last paragraph shows the author’s difficulty understanding how the
prediction was made. “But how was the prediction of mirror-image pictures made in the
first place? Was de la Roche’s prediction of mirror-image photography just a lucky
guess? Or was the Daguerreotype a picture-perfect case of life imitating art?”
B) Incorrect. In lines 14-15, the author writes, “Others became Daguerreotypists, now
known as photographers.” Here the author states that Daguerreotypists are the same as
photographers, so she has no difficulty comparing them.
C) Incorrect. Although may be true that the phrase is inaccurate, there was no mention that
the term “mirror with a memory” was inaccurate.
D) Incorrect. In lines 12-14, the author writes, “It was so successful, in fact, that French
newspapers said the French public had an illness called Daguerreotypomania!
Daguerreotypes were inexpensive and were suitable for portraiture,” which shows that
the French public liked Daguerretypes because they were inexpensive and suitable for
portraiture.
E) Incorrect. In lines 14-15, the author writes, “Others (other portrait artists) became
Daguerreotypists, now known as photographers.” There was no mention of the difficulty
of acquiring photography skills.
englishforeveryone.org Name________________
Date________________

Intermediate Critical Reading – The Robin
The North American Robin is usually called the “robin.” The robin is a bird. The male robin
has a bright red breast. The robin has a cheerful song.
Robins live throughout North America. They live in Canada. They live in Alaska. They live
in the 48 lower states. In the winter, robins go south. In the winter, some robins go to central
5 Mexico. Some robins go to the Pacific coast.
Robins eat during the day. Robins eat beetles. Robins eat berries. Robins eat fruits. Robins
eat about 40% invertebrates like beetles and grubs. Robins eat about 60% fruits and berries. Mother
robins find food for baby robins. Father robins find food for baby robins. Baby robins eat worms.
Baby robins eat other soft-bodied animals too.
10 Squirrels prey on robin eggs and baby robins. Snakes prey on baby robins and robin eggs.
Some other birds prey on baby robins and robin eggs. Blue jays prey on baby robins and robin eggs.
Crows prey on baby robins and robin eggs. Hawks prey on adult robins. Cats prey on adult robins.
Snakes prey on adult robins.
To stay safe, robins stay vigilant. Robins pay close attention to their surroundings. When
15 they gather in groups, they watch other robins for signs of danger. Robins give loud warning calls
when a dangerous predator approaches. Although robins are territorial, they band together for
protection from predators.
The robin is a sign a spring. Some people think that seeing the first robin of springtime brings
good luck.
Questions
1. According to the passage, which of the following statements is/are true?
I) Some robins go to South America during the winter.
II) The female robin has a bright red breast.
III) Cats hunt adult robins.

A) I only
B) II only
C) III only
D) I and II only
E) II and III only
2. What is the main idea of the fourth paragraph?
A) Squirrels hunt baby robins.
B) Other animals prey on robins.
C) Adult robins do not care for their young.
D) Crows and hawks hunt adult robins.
E) Other birds steal robin eggs.
3. In line 14, vigilant most closely means
A) attentive
B) dangerous
C) noisy
D) safe
E) territorial
Answers and Explanations

1. The correct answer is C.


I) Incorrect. Lines 4-5 state that “In the winter some robins go to central Mexico.
Some robins go to the Pacific coast.” While some robins may go to Central
America during the winter, there is no information in the passage about Central
America.
II) Incorrect. Lines 1-2 state that “The male robin has a bright red breast.” There is no
mention of the coloring of the female robin.
III) Correct. Line 13 states that “Cats prey on adult robins.” To prey on means to hunt,
so it is true that cats hunt adult robins.

A) I only
B) II only
C) III only
D) I and II only
E) II and III only

2. The correct answer is B.


A) Incorrect. While line 10 states that “Squirrels prey on robin eggs and baby robins,” the
paragraph goes on to list other animals which prey on, or hunt, robins and their eggs. So
the main idea does not center on squirrels, but rather on animals which prey on robins
and their eggs.
B) Correct. Lines 10-12 list the animals which prey on baby robins and robin eggs. Lines
12-13 list animals which prey on adult robins. Therefore, the entire paragraph is centered
on animals which prey on robins.
C) Incorrect. While lines 10-12 list animals which prey on baby robins and robin eggs, and
lines 12-13 list animals which prey on adult robins, there is no mention that the adult
robins do not care for their young.
D) Incorrect. Lines 12-13 state that “Crows prey on baby robins and robin eggs. Hawks prey
on adult robins.” However, there is no mention that crows prey on adult robins or that
hawks prey on baby robins and robin eggs.
E) Incorrect. While lines 10-12 state that some animals prey on baby robins and robin eggs,
the main idea does not center on the robin eggs, but rather on the animals which prey on
robins.

3. The correct answer is A.

A) Correct. According to line 14, “To stay safe, robins are vigilant. Robins pay close
attention to their surroundings.” Vigilant robins are robins which pay close attention to
their surroundings. To pay attention means to be attentive. Therefore, vigilant must
mean attentive.
B) Incorrect. According to lines 14-16, “To stay safe, robins are vigilant. Robins pay close
attention to their surroundings. When they gather in groups, they watch other robins for
signs of danger.” Since vigilant robins watch for signs of danger, it stands to reason that
the word vigilant does not mean dangerous.
C) Incorrect. According to lines 14-16, “To stay safe, robins are vigilant. Robins pay close
attention to their surroundings. When they gather in groups, they watch other robins for
signs of danger. Robins give loud warning calls when a dangerous predator approaches.”
Although vigilant robins do give loud or noisy warning calls in when there is a dangerous
predator in the area, the other sentences in the paragraph center on the attentiveness, the
watchfulness, and the awareness of danger that robins possess. So their noisiness seems
to stem from their vigilance, but noisy does not mean the same as vigilant.
D) Incorrect. According to line 14, “To stay safe, robins are vigilant.” While vigilance and
safety seem to go together, they do not have the same meaning. So the word safe does
not mean the same as vigilant.
E) Incorrect. The paragraph states, “To stay safe, robins are vigilant. … Although robins are
territorial, they band together for protection from predators.” While banding together for
protection is a way of staying safe or to find protection from predators, robins are also
said to be territorial. Being territorial seems to mean something very different from
banding together. Therefore, if birds are vigilant for safety and if they also band together
for safety, vigilant cannot mean the same as territorial.
englishforeveryone.org Name________________
Date________________


Advanced Critical Reading - Biomimetics
Biomimetics is the billion–dollar industry which draws inspiration from nature to solve
problems in engineering, medicine, and other fields. While human beings have always had a
natural propensity to observe and try to copy the ingenuity of nature, it is only recently that
biomimetics has taken off as a field of its own.
5 The Renaissance genius Leonardo da Vinci dedicated himself to observing and recording
natural phenomena half a millennium ago. His superb graphic renditions of plausible flying
machines are based on his direct observations of birds in flight. His renowned Codex Atlanticus,
along with smaller codices, includes renderings of animal musculature, revealing a fascination
with movement. His erudite studies of flowing hair and water manifest his contemplation of the
10 movement of waves in nature. However, without modern technology, early inventors like da
Vinci were unable to implement their biomimetic insights.
On the other hand, contemporary biomimeticists have access to the technology and
nanotechnology essential to the industry. One tool of enormous utility to today’s biomimetics is
the electron microscope. Being able to see and comprehend the nanoscale construction of natural
15 structures is crucial to synthesizing those miniscule formations.
One person who has taken on the challenge of biomimetics is Robert Fearing, a professor
of electrical engineering at the University of California, Berkeley. A modern–day echo of da
Vinci’s work, Fearing’s current challenge is to create a bio–robotic fly which is small, swift, and
maneuverable enough to deploy on surveillance or rescue missions. Fearing does not aspire to
20 replicate the fly. Rather, he hopes to isolate the enigmatic natural structures which give it flight,
and perhaps find a simpler solution than the 20 muscles which power a fly’s wing. Those 20
muscles allow it to make a 90–degree turn from straight–line flight in under 50 milliseconds,
something even the most advanced planes are not able to accomplish. “Some things are just too
mysterious and complicated to be able to replicate,” says Fearing.
25 Engineer Anthony Brennan, professor in the University of Florida’s Materials Science
and Engineering Department, learned that the pattern of miniscule diamond shapes on shark’s
skin inhibit the growth of algae and other organisms. He created a surface nanotechnology
comprised of billions of microscopic diamond–shaped bumps which repel bacteria and other
microorganisms. His biomimetic plastic wrap is being used in hospitals on typically bacteria–
30 laden surfaces such as light switches.
According to MIT chemical engineer Robert Cohen, “The natural structure provides a
clue to what is useful in a mechanism. But maybe you can do it better.” Cohen recently utilized
nanotechnology to mimic the scales of a desert lizard to produce a water collection device. In a
National Geographic interview, Cohen says, “Looking at pretty structures in nature is not
35 sufficient. What I want to know is, can we actually transform these structures into an
embodiment with true utility in the real world?”
Leonardo would have met today’s technologies with a receptive mind. In his own time,
he was heralded primarily not as an artist, but as an engineer. His orientation as a biomimeticist
might be summed up in his own quotation concerning nature: “Human subtlety will never devise
40 an invention more beautiful, more simple or more direct than does nature because in her
inventions nothing is lacking, and nothing is superfluous.”
Questions
1. According to the passage, how do today’s biomimeticists differ from Leonardo da Vinci?
I) Leonardo did not have the technology to construct his machines when he
envisioned them.
II) Today’s biomimeticists aspire to improve upon nature, whereas Leonardo
considered nature to be ultimately better than any man–made invention.
III) Today’s biomimeticists have conceived of replacing many natural structures with
man–made structures.

A) I only
B) II only
C) III only
D) I and II only
E) II and III only

2. How is Fearing’s current challenge an echo of da Vinci’s work?


A) Fearing is attempting to create a flying machine based on a natural structure.
B) Fearing hopes to improve upon the fly’s movement mechanism.
C) Fearing aspires to create a miniscule technology.
D) Fearing hopes to simplify modern flight technology.
E) Fearing is working on electricity.

3. In line 20, enigmatic most closely means

A) discrete
B) strategic
C) pragmatic
D) inscrutable
E) determinate

4. Based on the passage, which modern biomimeticist holds views most similar to those of
Leonardo as exemplified in lines 39–41, and why?
A) Robert Fearing –– because he wants to simplify the natural structures that give the fly
flight
B) Robert Fearing –– because he believes that some things are too complicated to be able to
replicate
C) Anthony Brennan –– because he replicated a structure found in nature
D) Anthony Brennan – because he created a surface technology
E) Robert Cohen –– because he says, “maybe you can do it better.”
Answers and Explanations
1. The correct answer is D.
I) Correct. Leonardo did not have the technology to construct his machines when he
envisioned them.
II) Correct. Today’s biomimeticists aspire to improve upon nature, whereas Leonardo
considered nature to be ultimately better than any man–made invention.
III) Incorrect. Today’s biomimeticists have not conceived of replacing many natural
structures with man–made structures.

2. The correct answer is A.


A) Correct. Fearing is attempting to create a flying machine based on a natural structure, just
as Leonardo wanted to build a flying machine based on the flight of birds.
B) Incorrect. Fearing hopes to improve upon the fly’s movement mechanism, however da
Vinci did not hope to improve on nature.
C) Incorrect. Fearing aspires to create a miniscule technology, but da Vinci did not aspire to
create a small technology.
D) Incorrect. Fearing hopes to simplify the fly’s natural flight mechanism.
E) Incorrect. There is no mention that Fearing is working on electricity. Fearing is an
electrical engineer.

3. The correct answer is D.

A) Incorrect. Discrete means separate. Since Fearing hopes to isolate the structures, or make
them separate, discrete cannot be correct.
B) Incorrect. Strategic means planned. Since fearing hopes to isolate these structures, he
does not yet know whether or not they are planned, so strategic cannot be correct.
C) Incorrect. Pragmatic means practical. Since Fearing hopes to simplify these structures, he
seems to believe they are not practical or pragmatic, so pragmatic cannot be correct.
D) Correct. Inscrutable means puzzling. Fearing is hoping to isolate the structures, so he
seems to believe they are puzzling, so enigmatic is correct.
E) Incorrect. Determinate means distinct. Since Fearing hopes to isolate the structures, or
make them determinate or distinct, determinate cannot be correct.

4. The correct answer is C.


A) Incorrect. Robert Fearing –– because he wants to simplify the natural structures that give
the fly flight, whereas da Vinci believed that nature had the simplest answers.
B) Incorrect. Robert Fearing –– because he believes that some things are too complicated to
be able to replicate, whereas da Vinci believed that nature had the simplest –– not
complicated – answers.
C) Correct. Anthony Brennan –– because, like da Vinci, he replicated a structure found in
nature
D) Incorrect – There is no mention that Leonardo created a surface technology.
E) Incorrect. Robert Cohen –– because he says, “maybe you can do it better,” whereas da
Vinci thought nature had the best answers.
englishforeveryone.org Name________________
Date________________


Advanced Critical Reading – Colony Collapse
According to a 2007 statement before the Subcommittee on Horticulture and Organic
Agriculture Committee of the U.S. House of Representatives by May R. Berenbaum, Professor
and Head, Department of Entomology at the University of Illinois Urbana-Champaign,
pollination by native bees was recently estimated to be worth 3 billion dollars annually in the
5 United States. Other sources estimate pollination to be a 15 billion dollar industry worldwide.
Moreover, approximately 3/4 of the 250,000 + species of flowering plants rely on their insect
partners – pollinators – for the vital natural process which transfers pollen grains to receptive
female plants to effect fertilization.
However, up to 80% of honeybees in 35 states have suddenly, inexplicably vanished,
10 baffling entomologists, beekeepers, farmers, and governments. About 10% of 2.4 million hives
disappeared in the U.S. in late 2006 and early 2007 in an enigmatic calamity that has been named
“colony collapse disorder,” or CCD. This mysterious killer has wiped out hives in the U.S. and
overseas – in Italy, Poland, Portugal, Central, South America, and Croatia. Some have theorized
that cell phones disrupt the ability of the insects to navigate – the bees leave the hive, never to
15 return. Others have surmised that a bee virus corresponding to HIV in humans has somehow
inculcated itself into the honeybee population. Still others have laid the blame on pesticides,
most notably a neurotoxin called imidacloprid – IMD, which has been shown by French
researchers to cause disorientation among honeybees. IMD also affects the ability of bees to
forage, affecting hive activity. While most French and some American beekeepers consider IMD
20 to be a major suspect in CCD, their opinion is not corroborated by American researchers. U.S.
scientists continue to pursue the cause, or amalgam of causes, of the disorder. On September 7,
2007, American researchers reported in the journal Science that a newly discovered virus, the
Israeli acute paralysis virus, may be a suspect in the catastrophic loss of honeybees. Meanwhile,
U.S. beekeepers are urgently requesting expeditious resolution to the crisis in the form of
25 emergency research funding.
The impact of CCD is almost unfathomable. One–third of all food is provided by the
pollination of honeybees. In a world without honeybees, there would be almost no fruits or
vegetables, except for those hand–pollinated by humans. At their current rate of disappearance,
honeybees will cease to exist by the year 2035.
Questions
1. It can be inferred from the passage that the U.S. pollination industry is
A) the top industrial pollinating nation in the world.
B) the second pollinating nation in the world.
C) among the top three pollinating nations in the world.
D) among the top four pollinating nations in the world.
E) among the top five pollinating nations in the world.

2. In line 20, corroborated most closely means

A) disputed.
B) omitted.
C) corrected.
D) supported.
E) researched.

3. Based on the passage, what does CCD cause bees to do?

A) Bees no longer leave their hives.


B) Bees bring a virus back to the hive.
C) Bees get lost and stop pollinating.
D) Bees become very active and exhausted.
E) Bees suddenly, inexplicably attack their hive mates.
Answers and Explanations

1. The correct answer is A.


A) Incorrect. According to lines 4–6, pollination by native bees was estimated to be worth 3
billion dollars annually in the United States in a 15 billion dollar industry worldwide.
Since 15 billion divided by 3 billion is 5, it can be inferred that the United States is in the
top five industrial pollinating nations. So, although it might be the top pollinating nation,
that information cannot be inferred from the passage.
B) Incorrect. According to lines 4–6, pollination by native bees was estimated to be worth 3
billion dollars annually in the United States in a 15 billion dollar industry worldwide.
Since 15 billion divided by 3 billion is 5, it can be inferred that the United States is in the
top five industrial pollinating nations. So, while it might be the second pollinating nation
in the world, that information cannot be inferred from the passage.
C) Incorrect. According to lines 4–6, pollination by native bees was estimated to be worth 3
billion dollars annually in the United States in a 15 billion dollar industry worldwide.
Since 15 billion divided by 3 billion is 5, it can be inferred that the United States is in the
top five industrial pollinating nations. So, while the U.S. might be in the top three
pollinating nations, that information cannot be inferred from the passage.
D) Incorrect. According to lines 4–6, pollination by native bees was estimated to be worth 3
billion dollars annually in the United States in a 15 billion dollar industry worldwide.
Since 15 billion divided by 3 billion is 5, it can be inferred that the United States is in the
top five industrial pollinating nations. So, while the U.S. might be in the top four
pollinating nations, that information cannot be inferred from the passage.
E) Correct. According to lines 4–6, pollination by native bees was estimated to be worth 3
billion dollars annually in the United States in a 15 billion dollar industry worldwide.
Since 15 billion divided by 3 billion is 5, it can be inferred that the United States is in the
top five industrial pollinating nations.

2. The correct answer is D.

A) Incorrect. Disputed means argued. Lines 20–21 state that U.S. scientists continue to
pursue the cause, or amalgam of causes, of the disorder, because the results of the French
are not corroborated by them. This indicates that they would argue with or dispute the
French diagnosis. So the correct answer cannot be disputed.
B) Incorrect. Omitted means left out. Since lines 20–21 state that U.S. scientists continue to
pursue the cause, or amalgam of causes, of the disorder, it seems that the singular
diagnosis of the French has been omitted, so omitted cannot be the correct answer.
C) Incorrect. Corrected means made accurate. Since lines 20–21 state that U.S. scientists
continue to pursue the cause, or amalgam of causes, of the disorder, it seems that the
Americans are correcting the diagnosis of the French. So corrected cannot be the correct
answer.
D) Correct. Supported means agreed with. Since lines 20–21 state that U.S. scientists
continue to pursue the cause, or amalgam of causes, of the disorder, it seems that the
results of the French are not supported or corroborated by them.
E) Incorrect. Researched means studied. Since lines 20–21 state that U.S. scientists continue
to pursue the cause, or amalgam of causes, of the disorder, it is clear that U.S. scientists
are researching the causes of the disorder. So researched cannot be the correct answer.
3. The correct answer is C.
A) Incorrect. Lines 14–15 state that the bees leave the hive, never to return. So it cannot be
correct that the bees do not leave the hive.
B) Incorrect. While a virus might be the cause, there is no mention that the bees bring a virus
back to the hive. According to lines 15–16, others have surmised that a virus
corresponding to HIV has somehow inculcated itself into the honeybee population. Since
the passage states that the researchers have surmised it, they are not sure that a virus is
the cause, so this cannot be the correct answer.
C) Correct. Lines 14–15 state that the bees leave the hive, never to return. That, with the
information from lines 16–20 – that a neurotoxin called imidacloprid – IMD – has been
shown by French researchers to cause disorientation among honeybees. IMD also affects
the ability of bees to forage, affecting hive activity. Since most French and some
American beekeepers consider IMD to be a major suspect in CCD, these must be the
symptoms that are exhibited by the bees.
D) Incorrect. While IMD affects the ability of bees to forage, affecting hive activity, as
stated in lines 18–19, there is no mention that the bees themselves become active.
E) Incorrect. According to line 9, up to 80% of honeybees in 35 states have suddenly,
inexplicably vanished. That means they have disappeared. However, they have not
suddenly, inexplicably attacked their hive mates.
englishforeveryone.org Name________________
Date________________


Advanced Critical Reading – Columbian Exchange
The Columbian Exchange was the “exchange of plants, animals, foods, human
populations (including slaves) communicable diseases, and ideas between the Eastern and
Western hemispheres that occurred after 1492,” according to Wikipedia. The term “Columbian
Exchange,” coined in 1972 by historian Alfred Crosby, took hold and became not only standard
5 shorthand for the phenomenon which it exemplified, but also a perspective for witnessing
societal and ecological events.
When Christopher Columbus made landfall with his crew in the Bahamas in October
1492, two worlds with separate evolutionary histories met. When Europeans began to settle
America’s east coast, they brought with them and cultivated familiar crops – wheat and apples –
10 as well as familiar weeds, such as dandelion and chickweed. In the 1600s, they introduced cattle
and horses, which flourished in the New World climate.
Devastating diseases were introduced to the American population which had no
resistance to them. John R. McNeill, professor of history at Georgetown University, points out
that “when the first inhabitants of the Americas arrived across the Bering land bridge between
15 20,000 and 12,000 years ago, they brought few diseases with them … they had no domesticated
animals, the original source of human diseases such as smallpox and measles. In addition, as they
passed from Siberia to North America, the first Americans had spent many years in extreme
cold, which eliminated many of the disease-causing agents that might have traveled with them.”
Consequently, between 1492 and 1650, over 90% of the Native American population died in
20 epidemic after epidemic of smallpox, measles, mumps, whooping cough, influenza, chicken pox,
and typhus. The loss of labor caused by pathogens indirectly led to the establishment of African
slavery among European immigrants in the Americas, resulting in the importation of malaria and
yellow fever from Africa, causing even more destruction of the Native American population.
The export of American flora and fauna did not revolutionize the Old World as the influx
25 of European agriculture altered the New World ecosystem. According to Crosby, “the New
World’s great contribution to the Old is in crop plants. … Maize, white potatoes, sweet potatoes,
various squashes, chiles, and manioc” augmented and invigorated the European cuisine. Very
few New World creatures traversed the ocean –– the muskrat, the gray squirrel, and a few others,
but they did not precipitate large scale changes in Old World ecosystems.
30 Although some diseases made the ocean voyage from New World to Old, they did not
have appreciable effects on the European population. Crosby stated that, although some deaths
were attributed to ailments from America, “the total is insignificant compared to Native
American losses to smallpox alone.”
In Crosby’s original work, he eschewed ideological statements. He reminded his readers
35 that neither the Old nor New World was inferior or superior to the other; the encounter between
two worlds was fundamentally an exchange. By 1988, he summarized his long view of the
encounter in this way: “My point is … that the impact of the Encounter is so massive that we
should consider it with the same sense of scale as we do events connected with the endings and
beginnings of the geological periods and eras and their influence on the direction of evolution on
40 the planet.”
Questions
45
1. Which of the following best describes the author’s view of the Columbian Exchange?
I) Neither the Old World nor the New World was superior to the other.
II) The New World experienced the brunt of the encounter between the Old and New
Worlds.
III) The encounter between the Old and New Worlds was fundamentally an even
exchange.

A) I only
B) II only
C) III only
D) I and II only
E) II and III only

2. It can be inferred from the passage that


A) Slaves brought to American from Africa had more resistance to European diseases than
Native Americans did.
B) New World creatures were unable to thrive in the climate of the Old World.
C) New World pathogens had no effect on the people of the Old World.
D) Most human diseases were introduced to humans by animal populations.
E) Europeans had more resistance to European diseases than Africans did.

3. In line 34, eschewed most closely means

A) espoused
B) avoided
C) employed
D) created
E) discovered
Answers and Explanations
1. The correct answer is D.
I) Correct. In lines 34–35, “In Crosby’s original work, he eschewed ideological statements.
He reminded his readers that neither the Old nor New World was inferior or superior to
the other; …”
II) Correct. According to lines 12–13. “Devastating diseases were introduced to the
American population which had no resistance to them.” According to lines 19–23,
“between 1492 and 1650, over 90% of the Native American population died in epidemic
after epidemic of smallpox, measles, mumps, whooping cough, influenza, chicken pox,
and typhus. The loss of labor caused by pathogens indirectly led to the establishment of
African slavery among European immigrants in the Americas, resulting in the
importation of malaria and yellow fever from Africa, causing even more destruction of
the Native American population.” And in lines 30–34, “Although some diseases made the
ocean voyage from New World to Old, they did not have appreciable effects on the
European population. Crosby stated that, although some deaths were attributed to
ailments from America, ‘the total is insignificant compared to Native American losses to
smallpox alone.’”
III) Incorrect. According to lines 35–36, “the encounter between two worlds was
fundamentally an exchange.” However, it is not stated that the exchange was an even
exchange.

2. The correct answer is A.

A) Correct. According to lines 19–20, “…between 1492 and 1650, over 90% of the Native
American population died in epidemic after epidemic of smallpox, measles, mumps,
whooping cough, influenza, chicken pox, and typhus.” This shows that Native Americans
had little resistance to European diseases. According to lines According to lines 21–22,
“The loss of labor caused by pathogens indirectly led to the establishment of African
slavery among European immigrants in the Americas, resulting in the importation of
malaria and yellow fever from Africa, causing even more destruction of the Native
American population.” .” Since the African slaves were able to work in conditions which
caused over 90% of the Native American population to die, the African slaves must have
had more resistance to European diseases than the Native Americans did.
B) Incorrect. According to lines 27–29, “Very few New World creatures traversed the ocean
–– the muskrat, the gray squirrel, and a few others, but they did not precipitate large scale
changes in Old World ecosystems.” This shows that the animals traveled to the Old
World and, since they did not make large–scale changes, probably made small–scale
changes in Old World ecosystems. Had they not thrived, they would not have made any
changes in the Old World ecosystems at all.
C) Incorrect. According to lines 31–33, “Crosby stated that, although some deaths were
attributed to ailments from America, ‘“the total is insignificant compared to Native
American losses to smallpox alone.”’ So it is not correct that New World pathogens had
no effect on the people of the Old World.
D) Incorrect. It is not possible to determine from this reading whether most human diseases
were introduced animal populations. The statement in lines 15–16 that “they had no
domesticated animals, the original source of human diseases such as smallpox and
measles” does not establish that most human diseases were introduced by animal
populations.
E) Incorrect. It is not possible to determine from this reading whether Europeans had more
resistance to European diseases than Africans did. The statement “The loss of labor
caused by pathogens indirectly led to the establishment of African slavery among
European immigrants in the Americas, resulting in the importation of malaria and yellow
fever from Africa, causing even more destruction of the Native American population”
does not establish that either Europeans or Africans had more resistance to European
diseases.

3. The correct answer is B.

A) Incorrect. He did not espouse or adopt ideological statements, which are statements of a
specific doctrine or ideology. Rather, as pointed out in lines 34–35, “he reminded his
readers that neither the Old nor New World was inferior or superior to the other.”
B) Correct. He eschewed or avoided ideological statements, which are statements of a
specific doctrine or ideology. As pointed out in lines 34–35, “he reminded his readers that
neither the Old nor New World was inferior or superior to the other.”
C) Incorrect. He did not employ or use ideological statements, which are statements of a
specific doctrine or ideology. Rather, as pointed out in lines 34–35, “he reminded his
readers that neither the Old nor New World was inferior or superior to the other.”
D) Incorrect. He did not create or invent ideological statements, which are statements of a
specific doctrine or ideology. Rather, as pointed out in lines 34–35, “he reminded his
readers that neither the Old nor New World was inferior or superior to the other.”
E) Incorrect. He did not discover ideological statements, which are statements of a specific
doctrine or ideology. Rather, as pointed out in lines 34–35, “he reminded his readers that
neither the Old nor New World was inferior or superior to the other.” Instead, as pointed
out in lines 36–40, “he summarized his long view of the encounter in this way: ‘“My
point is … that the impact of the Encounter is so massive that we should consider it with
the same sense of scale as we do events connected with the endings and beginnings of the
geological periods and eras and their influence on the direction of evolution on the
planet.”’
englishforeveryone.org Name________________
Date________________


Advanced Critical Reading - Ethanol
Ethanol (CH3CH2OH; which is also called ethyl alcohol, grain alcohol, and EtOH) is a
clear, colorless liquid. It is a renewable biofuel made from starch and sugar–based crops like
corn grain and sugar cane or from cellulosic feedstocks like grass, wood, or recycled
newspapers. Ethanol is a high–octane biofuel which performs so splendidly in internal
5 combustion engines that early automakers presumed it would be the world’s chief fuel.
American proponents of ethanol fuel highlight two principal advantages: its
environmental impact and its energy security benefits.
The adoption of ethanol reduces noxious emissions such as carbon monoxide (CO) and
pollutants from internal combustion engines; hence, it is appreciably less deleterious to the
10 environment than gasoline. Ethanol made from corn has been shown to reduce harmful emissions
by up to 13%, whereas ethanol made from cellulosic materials reduces dangerous emissions by
as much as 88%.
Ethanol is a renewable biofuel; in only six months a new crop can be grown, harvested,
and converted to fuel, so it is profitable for rural crop–producing economies. In addition, it keeps
15 engines clean and can be used in gasoline engines with no modifications when combining gas
with up to 10% ethanol. It can be used in specially modified vehicles called “flexible–fuel” or
“flex–fuel” vehicles in concentrations of up to 85%. Gasoline combined with 85% ethanol is
generally referred to as “E85.” Higher ratios of ethanol in the fuel mixture result in less reliance
on fossil fuels, so there is less dependence on imports.
20 American opponents of ethanol fuel point to three disadvantages: its price fluctuations, its
energy level, and its availability.
The price of ethanol fluctuates on a different cycle than gasoline; therefore, at times
ethanol is more expensive than gasoline, and at times it is cheaper. Another drawback of ethanol
is that it contains less energy per gallon than gasoline; even when it is cheaper per gallon than
25 conventional fuel, it does not take the vehicle as far as a gallon of gas. A car’s fuel economy with
ethanol can be expected to be 20–30% less than a vehicle which burns gasoline. So the
occasional cheaper price is offset by the lower energy yields. In addition, ethanol is not as widely
distributed as gasoline. It is readily available only in the Midwest; other areas have limited
ethanol infrastructure.
30 The Obama administration is working on expanding the ethanol infrastructure. In a recent
interview with 15 newspaper editors, President Obama characterized the U.S. position on
biofuels in this way: “Our challenge, I think, is to see our current ethanol technology as a bridge
to the biofuels technologies of the future. And that’s what we want to invest in, and that’s what
I’ll be directing my Department of Agriculture to focus on.”
Questions
1. It can be inferred from the passage that which of these statements about ethanol is/are true?
I) Burning ethanol made from wood produces less CO than burning ethanol made
from corn.
II) Burning ethanol made from grain produces less CO than burning gasoline.
III) Burning ethanol made from newspapers produces less CO than burning ethanol
made from grass.

A) I only
B) II only
C) III only
D) I and II only
E) II and III only

2. The author’s primary purpose is to


A) describe and define ethanol.
B) compare 2 types of ethanol.
C) support the adoption of ethanol.
D) explain advantages and disadvantages of gasoline.
E) explain advantages and disadvantages of ethanol.

3. In line 9, deleterious most closely means

A) dangerous, because it harms the environment less than gas.


B) beneficial, because it helps the environment less than gas.
C) splendid, because it performs less splendidly than gas.
D) unreliable, because it is less unreliable than gas.
E) expensive, because it is less expensive than gas.
Answers and Explanations
1. The correct answer is D.
I) Correct. Corn is a sugar–based crop, and wood is a cellulosic feedstock, according to line
4. According to lines 10–12, ethanol made from corn reduces harmful emissions by up to
13%, and ethanol made from cellulosic materials reduces harmful emissions by as much
as 88%. According to line 8, one of the noxious (harmful) emissions is carbon monoxide
(CO.) Therefore, burning ethanol made from wood produces less CO than burning
ethanol made from corn.
II) Correct. According to lines 8–10, ethanol reduces noxious emissions such as carbon
monoxide (CO) and pollutants from internal combustion engines; hence, it is appreciably
less deleterious to the environment than gasoline. According to lines 11–13, ethanol made
from corn reduces harmful emissions by up to 13%.
III) Incorrect. According to lines 3–4, grass, wood, and recycled newspapers are cellulosic
materials. And while, according to lines 11–12, using ethanol made from cellulosic
materials reduces harmful emissions by up to 88%, there is no comparison made among
the various cellulosic materials.

2. The correct answer is E.


A) Incorrect. While the author describes and defines ethanol in lines 1–6, she goes on to
explain the advantages and disadvantages of using it.
B) Incorrect. While the author makes several comparisons between sugar–based ethanol and
cellulosic ethanol, in lines 2–4, lines 10–12 she goes on to explain the advantages and
disadvantages of using it.
C) Incorrect. While the author explains the advantages of adopting ethanol in lines 8–19, she
goes on to explain the disadvantages of using it.
D) Incorrect. While the author explains that ethanol is less harmful to the environment than
gasoline in lines 8–10 and that gas works better and is more widely available than ethanol
in lines 24–28, she does so only insofar as she is comparing the advantages and
disadvantages of ethanol.
E) Correct. The structure of the passage is revealed in lines 6–7, which explain what the
proponents of ethanol highlight as advantages, and in lines 20–21, which explain what
the opponents of ethanol point to as disadvantages.

3. The correct answer is A.

A) Correct. According to line 8, ethanol reduces noxious (harmful) emissions; according to


lines 10–12, ethanol made from corn reduces harmful emissions by up to 13%, and
ethanol made from cellulosic materials reduces harmful emissions by as much as 88%. So
ethanol is less harmful, or less dangerous, than gasoline.
B) Incorrect. According to line 8, ethanol reduces noxious (harmful) emissions; according to
lines 10–12, ethanol made from corn reduces harmful emissions by up to 13%, and
ethanol made from cellulosic materials reduces harmful emissions by as much as 88%. So
ethanol is more beneficial, not less beneficial, than gasoline.
C) Incorrect. According to lines 4–5, ethanol performs so splendidly in internal combustion
machines that carmakers thought it would be the chief fuel. According to line 15, it keeps
engines clean. And while lines 25–26 point out that a car’s fuel economy can be less with
ethanol than with gasoline, there is no mention that ethanol performs less splendidly than
gas.
D) Incorrect. According to lines 4–5, ethanol performs so splendidly in internal combustion
machines that carmakers thought it would be the chief fuel. According to line 25, ethanol
does not take the vehicle as far as a gallon of gas. But there is no mention that it is more
or less unreliable than gas.
E) Incorrect. According to lines 9–10, ethanol is appreciably less deleterious to the
environment than gasoline. The expense of gasoline would not cause the environment to
change. According to lines 22–23, the price of ethanol fluctuates on a different cycle than
gasoline; therefore, at ethanol is more expensive than gasoline, and at times it is cheaper.
So, ethanol is not less expensive than gas.
englishforeveryone.org Name________________
Date________________


Advanced Critical Reading - Generations
Authors William Strauss and Neil Howe are known for their theories about cycles of
generations in American history. In their seminal Generations, and the subsequent 13th Gen, and
The Fourth Turning, they explore the history of America from 1584 to the present as a repeating
cycle of 4 consecutive generational archetypes. They refer to each cycle of four generations as a
5 constellation, and they posit that each constellational era corresponds to “recurring types of
historical events” and moods. They state that adjacent generations do not live similar lives, and
that each generation ages as a singular cohort as time moves forward. If one were to place this
progression on a graph, the line would form a diagonal – which Strauss and Howe call the
“generational diagonal.” According to Strauss and Howe, each generation is comprised of people
10 who possess 1) common age, 2) common beliefs, and 3) perceived membership in the same
generation. A generation is approximately 22 years in length. Since a lifetime may reach 80–90
years, members of 4 generations are alive at one time.
The four generational archetypes identified by Strauss and Howe are Idealist, Reactive,
Civic, and Adaptive. Idealists are “increasingly indulged youths after a secular crisis,” come of
15 age “inspiring a spiritual awakening,” cultivate principle rather than practicality or pragmatism
in midlife, and emerge as “visionary elders.” Reactives grow up “underprotected and criticized
youths during a spiritual awakening,” mature into risk taking adults, unlike the preceding
generation at midlife, mellow into “pragmatic midlife leaders during a secular crisis,” and
become reclusive elders. Civics grow up “increasingly protected youths after a spiritual
20 awakening,” become “a heroic and achieving cadre of young adults,” build institutions as
midlifers, and “emerge as busy midlifers, attacked by the next spiritual awakening.” Adaptives
grow up as “overprotected and suffocated youths during a secular crisis,” unlike the previous
generation as young adults, become “risk–averse, conformist rising adults,” mature into
“indecisive arbitrator leaders during a spiritual awakening,” and become sensitive elders.
25 The authors believe that the archetypical generations have recurred in fixed order
throughout American history with one exception: following the Civil War, one type did not
appear. As each generational archetype shifts from one phase of life to the next, the succeeding
generations line up in a predictably recurring pattern. For example, when the Idealists are elders,
the Reactives are in midlife, the Civics are in rising adulthood, and the Adaptives are in youth.
30 The central role of the elders, aged 66–87, is that of stewardship, such as supervising and
mentoring. The central role of midlife, aged 44–65, is leadership, such as parenting and teaching.
The central role of rising adulthood is activity, such as working and starting families. The central
role of youth, aged 0 to 21 is dependence, such as growing and learning.

35
Questions
1. According to the passage, which of the following statements can be inferred?
I) At a time when a Reactive generation member is a pragmatic leader, an Idealist
generation member is a visionary elder.
II) At a time when a Civic generation member is a protected youth, a Reactive
generation member is a risk–taking adult.
III) At a time when an Adaptive generation member is a sensitive elder, a Civic
generation member is an institution–building midlifer.

A) I only
B) II only
C) III only
D) I and II only
E) II and III only

2. According to the passage, what happens to the Civic generation as its members enter midlife?
A) It is attacked by Idealists who are coming of age.
B) It is attacked by Idealists who are visionary elders.
C) It is attacked by Reactives who are pragmatic leaders.
D) It is attacked by Adaptives who are rising adults.
E) It is attacked by Adaptives who are youths.

3. In line 15, pragmatic most closely means

A) acting on the basis of principle.


B) behaving in a hermit–like way.
C) being in the final stage of life.
D) acting in a practical way.
E) behaving in a reckless way.
Answers and Explanations

1. The correct answer is D.


I) Correct. Since the Reactives are one generation younger than the Idealists, the
member of the Reactive generation are one life phase younger than the members of
the Idealist generation. Therefore, when the Idealists are elders, the Reactives are in
midlife (lines 28–29.)
II) Correct. Since the Civics are one generation younger than the Reactives, the
members of the Civic generation are one life phase younger than the members of
the Reactive generation. Therefore, when the Reactives are adults, the Civics are
youths (lines 28–29.)
III) Incorrect. Since the Adaptives are one generation younger than the Civics, the
members of the Adaptive generation are one life phase younger than the members
of the Civic generation. Therefore, when the Civics are midlifers, the Adaptives are
not elders; they are rising adults (lines 28–29.)

A) I only
B) II only
C) III only
D) I and II only
E) II and III only

2. The correct answer is A.


A) Correct. According to lines 20–21, Civics “emerge as busy midlifers, attacked by the next
spiritual awakening.” The generation which attacks them is the Idealist generation, which
comes of age “inspiring a spiritual awakening,” shown in lines 14–15.
B) Incorrect. According to lines 20–21, Civics “emerge as busy midlifers, attacked by the
next spiritual awakening.” The generation which attacks them is the Idealist generation,
which comes of age “inspiring a spiritual awakening,” shown in lines 14–15. When the
idealists age, they become “visionary elders,” as shown in line 16.
C) Incorrect. According to lines 20–21, Civics “emerge as busy midlifers, attacked by the
next spiritual awakening.” The generation which attacks them is the Idealist generation,
which comes of age “inspiring a spiritual awakening,” shown in lines 14–15. When the
Civics are in midlife, the Reactives are elders, as can be inferred from line 29. When “the
Reactives are in midlife, the Civics are in rising adulthood.” When the Reactives are at
midlife, they do not attack. Instead, they “mellow into ‘“pragmatic midlife leaders during
a secular crisis”’ according to line 18.
D) Incorrect. Although it can be inferred that when the Civics are midlifers, the Adaptives
are rising adults (line 29), the generation which attacks the Civics is the Idealist
generation, which comes of age “inspiring a spiritual awakening,” shown in lines 14–15.
E) Incorrect. Since the Adaptives are one generation younger than the Civics, the members
of the Adaptive generation are one life phase younger than the members of the Civic
generation. Therefore, when the Civics are midlifers, the Adaptives are not youths; they
are rising adults (lines 28–29.)

3. The correct answer is D.


A) Incorrect. Principle is a belief or set of beliefs. According to lines 14–15, Idealists `
“cultivate principle rather than practicality or pragmatism in midlife.” Therefore,
principle is not the same as pragmatism.
B) Incorrect. A hermit is a reclusive person, who tends avoid other people. According to
lines 18–19, Reactives are “pragmatic midlife leaders during a
secular crisis,” who become reclusive elders. Since they become reclusive (which means
hermit–like) elders after being pragmatic leaders, the word pragmatic cannot mean
behaving in a hermit–like way.
C) Incorrect. The final stage of life is old age. Since Reactives are “pragmatic midlife
leaders during a secular crisis” as stated in line 18, the word pragmatic cannot refer to
the final stage of life.
D) Correct. Acting in a practical way means acting in a way to get things done. According
to lines 14–15, Idealists “cultivate principle rather than practicality or pragmatism in
midlife.” So practicality is similar to pragmatism.
E) Incorrect. Behaving in a reckless way means behaving in a way that invites danger.
Reactives mature into risk taking adults, according to lines 16–17 and mellow into
“pragmatic midlife leaders during a secular crisis.” Since they move from being risk–
taking (reckless) adults to pragmatic midlife leaders, the word pragmatic cannot mean
behaving in a reckless way.
englishforeveryone.org Name________________
Date________________


Advanced Critical Reading - Hubble
The 32,000–word novella The Time Machine by H.G. Wells, published in 1895, is
generally credited with popularizing the idea of time travel by means of a time machine, a
vehicle which takes the occupant backward or forward in time. Dozens of sequels and
adaptations over the years have further promoted the notion. Indeed, Albert Einstein’s Theory of
5 Special Relativity lays the foundation for the possibility of time travel. So far, no one has
demonstrated the ability to travel in time. However, time machines have been constructed, and
they do allow glimpses into the past.
The most efficacious time machine currently in existence is the Hubble Telescope, named
after the American astronomer Edwin P. Hubble. Its capability to locate distant astronomical
10 targets and lock in on them, permitting their faint light to aggregate on its detectors, allows it to
peer far into the past. Light travels 186,000 miles per second. The Hubble Telescope has looked
back in time at 10,000 galaxies whose light left them billions of years ago. Therefore, utilizing
the telescope as time machine, astronomers are able to contemplate galaxies as they were eons
ago.
15 Although the telescope was launched into space in 1990, its inception was almost a half–
century earlier as astronomer Lyman Spitzer, Jr. mulled over the possibility of a large space
telescope in a 1946 report, “Astronomical Advantages of an Extra–Terrestrial Observatory.”
Because the earth is bathed in its constantly churning atmosphere, earth–based telescopes cannot
penetrate deep space; the atmosphere distorts the view. Telescopes were constructed on
20 mountains, but there was still no way to wholly escape the effects of the layers of gases
enveloping the earth.
During the 1960s, the Space Race between the then–Soviet Union and the United States
was accelerating. The National Aeronautics and Space Administration (NASA) was established.
Funds for space endeavors were abundant, and plans for a large space telescope, by then
25 designated the LST, were underway. The designs called for a 2.4–meter primary telescope mirror
which could be transported into space by one of NASA’s rockets. According to National
Geographic’s Imaging Space and Time, the resolving power of the deep space telescope would
be “equivalent to being able to distinguish the left and right headlights of a car in California seen
from New York, or features less than 1/30,000th the size of the full moon. This was at least a
30 tenfold increase over the atmospheric limit.”
One of the primary challenges involved in successfully transporting the telescope into
space was protecting the mirror from the jarring vibrations that occur during launch. It was
crucial that the mirror be able to withstand the shuttle’s vicissitudes as well as the volatile
atmospheric conditions found in space. If not, the precise shape of the mirror could be
35 compromised, and its imaging capability significantly weakened.
After the telescope had been launched, astronomers subsequently realized that the
primary mirror had not been ground correctly. A lens in the test instrument was about one
millimeter askew, which is large by optical standards. In 1993, space–walking astronauts
installed corrective lenses which improved the eyesight of the Hubble. In 2009, the corrective
40 lenses themselves were replaced with a supersensitive spectrograph with built–in corrective
lenses. The new spectrograph is expected to provide insight into the origins of stars and galaxies.
The successor to Hubble, the James Webb Space Telescope, is expected to be launched in
2014. It will observe only in infrared, so it will complement the Hubble Telescope, which
observes in the visible and ultraviolet light ranges.
45 Hubble currently has the capability to view galaxies that were formed 13.7 billion years
ago, long before humans existed, in an area called the Hubble Ultra Deep Field. Astronomers
aspire to see beyond the Hubble Ultra Deep Field to a time that is devoid of galaxies, a time
before galaxies had formed. If H.G. Wells was onto something in his novella, that time may be
close at hand. As one of the characters in the popular work asked, “If Time is really only a fourth
50 dimension of Space, why is it, and why has it always been, regarded as something different? And
why cannot we move in Time as we move about in the other dimensions of Space?”
Less than a decade after Wells’ novella, Einstein’s Special Theory Relativity seemed to
concur with Wells’ character by proposing that traveling through space at the speed of light
would alter time by causing it to dilate, raising the possibility of not merely glimpsing the past,
55 but perhaps traveling to it.

Questions
1. According to the passage, which of the following statements is/are true of the Hubble
Telescope?
I) It is unable to observe light on the infrared part of the spectrum.
II) It will be replaced by the James Webb Space Telescope in 2014.
III) It was initially constructed in 1946, but not launched until 1990.

A) I only
B) II only
C) III only
D) I and II only
E) II and III only

2. According to the passage, who had the idea for the Hubble Telescope?
A) H.G. Wells
B) Albert Einstein
C) Lyman Spitzer, Jr.
D) Edwin P. Hubble
E) James Webb

3. In line 33, vicissitudes most closely means

A) long delays which may compromise the shuttle launch


B) toxic emissions which may cause corrosion around the mirror
C) sound waves which may penetrate the mirror
D) atmospheric conditions which may compromise the mirror
E) shaking and quivering which may cause changes in the mirror
4. In the context of the passage, which of the following best articulates the author’s opinion of
the inception of the Hubble?
A) It was a pipedream with little imminent chance of success.
B) It was a literary vehicle with little basis in reality.
C) It was an emergency response to the quickening Space Race.
D) It was based on a scientific proposition which was not proven.
E) It was a waste of time and money which were needed elsewhere.

5. The primary purpose of the passage is to

A) draw a comparison between H.G. Wells’ notion of time travel with Albert Einstein’s
Special Theory of Relativity.
B) discuss the construction of the Hubble Space Telescope as a tool for exploring deep
space.
C) examine difficulties which precipitated construction of corrective lenses for the
Hubble’s primary mirror.
D) describe the circumstances which underlay the mid–century national drive toward a large
space–based observatory.
E) dispute the argument that the Hubble Telescope functions as a modern–day time
machine.

6. It can be inferred that the author regards time travel as

A) an effective hook for a work of fiction, but an improbability in the reality of astronomy.
B) an interesting literary notion, but proven to be impossible by Einstein’s Special Theory.
C) a persuasive topic in fiction, as well as a hypothetical possibility in light of Einstein’s
Special Theory.
D) a ridiculous idea whose time has come and gone, as well as an astronomical
improbability.
E) the incoherent literary construction of a fictional author, with little relevance to today’s
scientific community.

7. It can be inferred from the passage that scientists believe that time is

A) a constant.
B) unidirectional.
C) a spatial dimension.
D) an impenetrable mystery.
E) an imaginary construction.
Answers and Explanations

1. The correct answer is A.


I) Correct. It is unable to observe light on the infrared part of the spectrum. It will be
replaced by the James Webb Space Telescope in 2014.
II) Incorrect. It will not be replaced by the James Webb Space Telescope in 2014; it
will be complemented by it.
III) Incorrect. It was initially constructed as part of the Space Race, but not launched
until 1990.

A) I only
B) II only
C) III only
D) I and II only
E) II and III only

2. The correct answer is C.


A) Incorrect. H.G. Wells wrote a novella about time travel.
B) Incorrect. Albert Einstein theorized about space and time.
C) Correct. Lyman Spitzer, Jr. wrote a 1946 paper about a space–based observatory.
D) Incorrect. The Hubble Telescope was named after Edwin P. Hubble.
E) Incorrect. The successor to the Hubble Telescope is named after James Webb.

3. The correct answer is E.

A) Incorrect. There was no mention of launch delays in the passage.


B) Incorrect. There was no mention of toxic emissions in the passage.
C) Incorrect. There was no mention of sound waves in the passage.
D) Incorrect. Although volatile atmospheric conditions were mentioned in the same
sentence, this factor that may occur in addition to vicissitudes experienced during launch.
E) Correct. Vicissitudes are jarring changes or difficulties. Therefore, shaking and quivering
which may cause changes in the mirror is the best answer. Even if you don’t know the
definition of this difficult GRE vocabulary word, you can infer its meaning from the
sentences which come before and after. The passage states that, “One of the primary
challenges involved in successfully transporting the telescope into space was protecting
the mirror from the jarring vibrations that occur during launch,” and if this was not
accomplished, then, “The precise shape of the mirror could be compromised, and its
imaging capability significantly weakened.”

4. The correct answer is A.

A) Correct. Astronomer Lyman Spitzer, Jr. mulled over the possibility of a large space
telescope in a 1946 report, as explained in lines 16–17. He did not plan the observatory. It
was a pipedream with little imminent chance of success.
B) Incorrect. It was a literary vehicle at the time of H.G. Wells, as shown in line 1. By the
1940s, it was a possibility, as shown in line 17.
C) Incorrect. It was built during the Space Race because money was available, as explained
in line 24.
D) Incorrect. It was based on scientific facts, as Spitzer explained in his 1946 report, line 17.
E) Incorrect. A successor telescope is being planned, which would not happen if it were a
waste of time of money, lines 42–43.

5. The correct answer is B.

A) Incorrect. Although the passage draws a comparison between H.G. Wells’ notion of time
travel with Albert Einstein’s Special Theory of Relativity, the passage goes on to discuss
the Hubble Space Telescope as a time machine.
B) Correct. The passage discusses the construction of the Hubble Space Telescope as a tool
for exploring deep space, such as the Hubble Ultra Deep Field and beyond.
C) Incorrect. The passage only briefly touches on the difficulties which precipitated
construction of corrective lenses for the Hubble’s primary mirror.
D) Incorrect. The passage does not describe the circumstances which underlay the mid
century national drive toward a large space–based observatory.
E) Incorrect. The passage does not dispute the argument that the Hubble Telescope functions
as a modern–day time machine.

6. The correct answer is C.

A) Incorrect. The author regards time travel as an effective hook for a work of fiction, but
hopes for its implementation in the reality of astronomy.
B) Incorrect. The author regards time travel as an interesting literary notion, but time travel
has not been proven to be impossible by Einstein’s Special Theory.
C) Correct. The author regards time travel to be a persuasive topic in fiction, as well as a
hypothetical possibility in light of Einstein’s Special Theory.
D) Incorrect. The author does not suggest that time travel is a ridiculous idea whose time and
come and gone, nor does the author suggest that is it an astronomical improbability. The
author says that H.G. Wells’ idea was popular, and expresses the hope of eventual
possible time travel.
E) Incorrect. The author does not suggest that time travel is the incoherent literary
construction with little relevance to today’s scientific community. The author says that
H.G. Wells’ idea was popular, and that the Hubble is, in fact, a time machine.

7. The correct answer is C.

A) Incorrect. Einstein’s Special Theory suggests that time can be manipulated through
speed of travel, as explained in lines 53–54, so time is not a constant.
B) Incorrect. Since time might be manipulated through speed of travel, as explained in lines
53–54, it is not necessarily considered unidirectional.
C) Correct. It is thought to be some sort of spatial dimension, alluded to in lines 46–54.
D) Incorrect. Einstein’s Special Theory suggests that it is not an impenetrable mystery, lines
4–5 and 53–54.
E) Incorrect. Scientists hope to be able to view beyond the Hubble Ultra Deep Field, which is
a time, as explained in lines 45–48. It is not an imaginary construction.
englishforeveryone.org Name________________
Date________________


Advanced Critical Reading – Intelligence Augmentation
The terms “intelligence augmentation” and “intelligence amplification” evoke images of
human beings with computer chips embedded in their skulls or bizarre accoutrements attached to
their heads. However, according to an article entitled Get Smart by Jamais Cascio, human
beings’ ability to augment their intelligence is precisely the prowess which has empowered us to
5 survive “a series of convulsive glacial events” evinced by the last ice age.
Neurophysiologist William Calvin asserts that the human species continues to evolve
cognitively and to create its own cognitive evolution in two basic ways: external and internal.
Cascio states that humans have been externally augmenting their intelligence for
millennia. By developing written language, we boosted our capacity to share information over
10 space and time. Other advancements, such as agricultural and industrial technologies, reduced
the exigencies of manual labor. Current external digital systems augment human intelligence by
allowing us to perform tasks that would be unfeasible with recourse only to the rational skills of
a singular human brain. Cascio cites as examples the “powerful simulations and massive data
sets (which) allow physicists to visualize, understand, and debate models of an 11–dimension
15 universe, real–time data from satellites, global environmental databases, and high–resolution
models (which) allow geophysicists to recognize the subtle signs of long–term changes to the
planet,” and similar man–made interactions which have the functional effect of augmenting
human intelligence. Conceivable potential software could incorporate individual “attention
filters” or “focus assistants” which would discern and highlight your individual preferences in a
20 computer display, permitting you to focus and direct your computer searches more efficiently
than you do now. It could incorporate individualized planning and foresight systems which could
allow people to play “what–if” with their life choices. Such systems could co–evolve with people
to produce intimate technologies which would become “something akin to collaborative
intuition,” through web–based information systems with personalized components, according to
25 Cascio.
Somewhat more problematic in social terms might be pharmacological intelligence
augmentation, evoking Brave New World nightmares – pharmaceutically placated people
tranquilized to zombie–like subservience to the collective and a central bureaucracy dedicated to
its own continued survival. However, as with external cognitive augmentation, the future has
30 arrived –– in the form of, for example, ADD drugs, pharmaceutical agents which mitigate sleep
disorders, and antidepressants, all of which enhance human problem–solving ability and
cognitive efficiency. According to Cascio, “people who don’t know about (such drugs) or don’t
want to use them will face stiffer competition from people who do. From the perspective of a
culture immersed in athletic doping wars, the use of such drugs may seem like cheating. From
35 the perspective of those who find they’re much more productive using this form of enhancement,
it’s no more cheating than getting a faster computer or a better education.”
Cognitive amplification, whether by external or internal means, may constitute evolution,
if Calvin’s assertion is correct. Some societies may readily embrace it, while others may shy
away. As science fiction writer William Gibson observes, “The future is already here; it’s just
40 unevenly distributed.”
Questions

1. The author is mainly concerned about


A) various dangers of intelligence augmentation.
B) the advantages of intelligence augmentation.
C) the basic methods of intelligence augmentation.
D) some scientists who are working on intelligence augmentation.
E) the differences between external and internal intelligence augmentation.

2. The author’s use of the phrase “somewhat more problematic in social terms” refers to
A) the difficulty of making cognitive enhancement widely available.
B) the difficulty of making pharmacological enhancement socially acceptable.
C) equalizing cognitive competitive advantages among social groups.
D) bureaucracies which hamper cognitive enhancement activities.
E) the relationship between external and internal intelligence augmentation.

3. As it is used in line 27, the word placated most closely means


A) deprived.
B) enhanced.
C) cured.
D) assisted.
E) quieted.

4. In the context of the passage, which of the following best articulates the author’s opinion?
A) Intelligence amplification by external means might be more difficult to achieve than by
internal means.
B) Cognitive augmentation does not really constitute evolution.
C) Some people consider intelligence enhancement to be a form of cheating.
D) External and internal intelligence enhancement might constitute evolution in cultures that
accept them.
E) Personalized software could be misused by a bureaucracy intent on its own continued
survival.

5. The primary purpose of the passage is to

A) describe different kinds of intelligence enhancement.


B) discuss society’s reactions to pharmacological cognitive augmentation.
C) examine the differences between external and internal intelligence enhancement.
D) dispel misgivings about humanity’s attempts at creating its own evolution.
E) illustrate the limitations of external intelligence augmentation.
Answers and Explanations

1. The correct answer is C.


A) Incorrect. Although the author alludes to dangers involved with intelligence
augmentation, she does not specifically mention any dangers. The ability to play “what–
if” with life choices, mentioned in line 23, could portend a dangerous situation. People’s
fears of a Brave New World situation, mentioned in lines 27–28, allude to dangers
discussed by Aldous Huxley in his novel entitled Brave New World. However, danger is
not the author’s main concern.
B) Incorrect. Although the author discusses the advantages of intelligence augmentation, this
is not her main concern. Lines 8–25 mostly discuss the methods of external intelligence
enhancement, and lines 29–32 discuss the advantages of internal intelligence
enhancement. These are interspersed with comments about fear related to intelligence
enhancement (lines 27–28) and some doubt regarding intelligence augmentation (33–34),
so the advantages of intelligence augmentation are not the author’s main concern.
C) Correct. In the second paragraph, the author introduces the topic of cognitive
augmentation by stating that the “neurophysicist William Calvin asserts that the human
species continues to evolve cognitively and to create its own cognitive evolution in two
basic ways: external and internal.”
D) Incorrect. Although several scientists are mentioned, the scientists are not the author’s
main concern. The author of Get Smart, Jamais Cascio, may or may not be a scientist.
Neurophysicist William Calvin is mentioned in the second paragraph when the author
introduces the topic. The only other person mentioned by name is science fiction writer
William Gibson. Geophysicists are mentioned in line 16, but the scientists involved in
intelligence augmentation are not the author’s main concern.
E) Incorrect. Although the author discusses both external and internal intelligence
augmentation, she does not say much about the differences between the two types of
cognitive enhancement. The only difference she mentions is in lines 26–27 – “Somewhat
more problematic in social terms might be pharmacological intelligence augmentation.”

2. The correct answer is B.

A) Incorrect. The author seems to say that pharmacological agents for intelligence
enhancement are already readily available. She says in lines 29–32, “as with external
cognitive augmentation, the future has arrived – in the form of, for example, ADD drugs,
pharmaceutical agents which mitigate sleep disorders, and antidepressants, all of which
enhance human problem–solving ability and cognitive efficiency.”
B) Correct. The author mentions social acceptance of intelligence augmentation in several
places in the passage. In lines 26–29, she says, “somewhat more problematic in social
terms might be pharmacological intelligence augmentation, evoking Brave New World
nightmares – pharmaceutically placated people tranquilized to zombie–like subservience
to the collective and a central bureaucracy dedicated to its own continued survival,”
which points up societal fear of pharmaceutical augmentation. In lines 32–34, she also
says, “people who don’t know about (such drugs) or don’t want to use them will face
stiffer competition from people who do. From the perspective of a culture immersed in
athletic doping wars, the use of such drugs may seem like cheating,” which shows a
moral resistance to pharmacological augmentation. Finally, in lines 38–39, the author
says, “some societies may readily embrace it, while others may shy away,” which points
to a larger societal aversion to pharmacological intelligence augmentation.
C) Incorrect. The author does mention competition between people who take advantage of
pharmacological intelligence boosts and people who do not. (lines 32–36) However, this
is not the subject that the author finds problematic. The problematic issue is overcoming
fear of pharmacological agents. In lines 26–29, she says, “Somewhat more problematic in
social terms might be pharmacological intelligence augmentation, evoking Brave New
World nightmares – pharmaceutically placated people tranquilized to zombie–like
subservience to the collective and a central bureaucracy dedicated to its own continued
survival,” which points up societal fear of pharmaceutical augmentation.
D) Incorrect. Although the author mentions bureaucracies, she does so in the context of a
nightmarish scenario in which the bureaucracies provide pharmaceutical agents in order
to placate the people, not to enhance their intelligence. In lines 26–29, she says,
“somewhat more problematic in social terms might be pharmacological intelligence
augmentation, evoking Brave New World nightmares – pharmaceutically placated people
tranquilized to zombie–like subservience to the collective and a central bureaucracy
dedicated to its own continued survival,” which points up societal fear of pharmaceutical
augmentation.
E) Incorrect. Although the author discusses both external and internal methods of
augmentation, she does not discuss the relationship between the two, except in lines 29–
30, when she says, “as with external cognitive augmentation, the future has arrived,” and
in the concluding paragraph when she says, “Cognitive amplification, whether by
external or internal means, may constitute evolution, if Cascio’s assertion is correct.”

3. The correct answer is E.

A) Incorrect. In lines 27–28, the statement “pharmaceutically placated people tranquilized to


zombie–like subservience to the collective” rules out the possibility that the word
placated could mean deprived. If they were pharmaceutically deprived, they would not be
tranquilized.
B) Incorrect. In lines 27–28, the statement “pharmaceutically placated people tranquilized to
zombie–like subservience to the collective” rules out the possibility that the word
placated could mean enhanced. If they were pharmaceutically enhanced, they would not
be tranquilized.
C) Incorrect. In lines 26–28, the statement “Somewhat more problematic in social terms
might be pharmacological intelligence augmentation, evoking Brave New World
nightmares – pharmaceutically placated people tranquilized to zombie–like subservience
to the collective” rules out the possibility that the word placated means cured, as it would
not be nightmarish to be cured.
D) Incorrect. In lines 27–28, the statement “pharmaceutically placated people tranquilized
zombie–like subservience to the collective” rules out the possibility that the word
placated could mean assisted. If they were pharmaceutically assisted, they would not
be tranquilized or zombie–like.
E) Correct. In lines 27–28, the statement “pharmaceutically placated people tranquilized
zombie–like subservience to the collective” points to the likelihood that the word
placated means something similar to tranquilized, or quieted.

4. The correct answer is B.

A) Incorrect. In lines 8–24, the author mentions many methods of external intelligence
augmentation, including writing, agricultural and industrial technologies, digital systems,
satellites, databases, attention filters, software, web–based information systems, and other
computer systems. In lines 30–31, she mentions only 3 kinds of pharmacological agents
to enhance intelligence. Therefore, external enhancements do not seem to be more
difficult to achieve than internal enhancements.
B) Incorrect. In lines 37–38, the author states that “cognitive amplification, whether by
external or internal means may constitute evolution if Calvin’s assertion is correct.” The
author, however, neither agrees nor disagrees that the assertion is correct.
C) Incorrect. In lines 32–34, the statement “According to Cascio, ‘“people who don’t know
about (such drugs) or don’t want to use them will face stiffer competition from people
who do. From the perspective of a culture immersed in athletic doping wars, the use of
such drugs may seem like cheating’” is not the author’s opinion. It is a quotation from
Cascio, the author of the Get Smart article.
D) Correct. In the concluding paragraph, the author states, “Cognitive amplification, whether
by external or internal means may constitute evolution, if Calvin’s assertion is correct.
Some societies may readily embrace it, while others may shy away.”
E) Incorrect. Rather than pointing to the possible misuse of personalized software, the
reference to personalized software is positive, in lines 23–26, “Such systems could co–
evolve with people to produce intimate technologies which would become “something
akin to collaborative intuition,” through web–based information systems with
personalized components, according to Cascio.”

5. The correct answer is A.

A) Correct. In the second paragraph, the author introduces the topic of cognitive
augmentation by stating that the “neurophysicist William Calvin asserts that the human
species continues to evolve cognitively and to create its own cognitive evolution in two
basic ways: external and internal.”
B) Incorrect. The author touches on the reaction of society to cognitive enhancement in lines
26–27 when she states, “Somewhat more problematic in social terms might be
pharmacological intelligence augmentation,” in line 34, “the use of such drugs may seem
like cheating,” and in lines 37–38 when she says, “Some societies may readily embrace
it, while others may shy away.” However, these ideas are not developed.
C) Incorrect. Although the author discusses both external and internal intelligence
augmentation, she does not say much about the differences between the two types of
cognitive enhancement. The only difference she mentions is in lines 26–27 – “Somewhat
more problematic in social terms might be pharmacological intelligence augmentation.”
D) Incorrect. The author mentions societal misgivings regarding cognitive enhancement in
lines 26–27 when she says, “Somewhat more problematic in social terms might be
pharmacological intelligence augmentation,” in line 34, “the use of such drugs may seem
like cheating,” and in lines 37–38 when she says, “Some societies may readily embrace
it, while others may shy away.” The author does not dispel these misgivings.
E) Incorrect. In lines 8–25, the author discusses external intelligence augmentation. She does
not mention limitations of any of the enhancements.
englishforeveryone.org Name________________
Date________________


Advanced Critical Reading - Salt
Salt is a mineral that consists mostly of sodium chloride (NACl). It is an essential nutrient
for animals, yet it is toxic to most plants. In her novel Tongue, author Kyung Ran Jo recounts
this legend about salt: “A long time ago, a princess told the king, ‘I love you as much as I love
salt.’ Believing it to be an insult, the king banished his daughter from his kingdom. But after a
5 long time, the king realized the value of salt and the depth of his daughter’s love for him.”
Saltiness is one of the basic tastes perceived by the tongue, making it an esteemed and
ubiquitous food flavoring. It also “retains vegetables’ vivid colors when parboiling, removes
astringency from salad greens, freezes ice cream, quickly cools boiling water, maintains the
freshness of cut flowers, removes stains on clothing, alleviates pain in your neck, is an ingredient
10 in soap,” according to Jo. Darlene McFarlane in her article “15 Household Uses for Table Salt”
recommends testing an egg’s freshness by placing it in a cup of salt water. An egg that floats is
not fresh. Ants will not venture onto a salt-covered surface, according to McFarlane, so she
suggests sprinkling it on windowsills and in doorways to repel them from your residence.
Salt’s historical distinction lies not so much in its taste or any of its aforementioned
15 amazing talents, however, as in its suitability as a preservative. Salt has been used as a food
preservative for centuries. One of the oldest documented saltworks is the Xiechi Lake near
Yuncheng in Shanxi, China. Salt was harvested from its surface as early as 6000 B.C. Salt, along
with salted birds and salt fish, was unearthed with funereal offerings in ancient Egyptian tombs
from the third millennium B.C. Less than half a century later, Egypt instituted exportation of salt
20 fish to the Phoenicians, who in turn traded Egyptian salt fish with their commercial partners
throughout North Africa, engendering the establishment of wide-ranging trade associations
throughout the Mediterranean region. Similarly, in the first millennium B.C., Celtic people
exchanged salted meat for wine and other luxury goods from ancient Greece and Rome. The
wide expanse of the Celtic salt trade is exemplified by the shared Celtic, Greek, and Egyptian
25 root word for salt, hal, which is iterated in the names of saltworks throughout the region: Halle
and Schwäbisch Hall in Germany, Halych in Ukraine, and Galicia in Spain.
Throughout history, salt has been deemed a precious commodity. In fact, the word
“salary” is derived from the Middle English salaire, from the Latin salarium, which means a
payment made in salt (sal) or for salt, from salarius which means “pertaining to salt.” Many
30 historians agree that the Latin word salarium is related to salt and soldiers, but stress that the
original association is unclear. Some surmise that soldiers were remunerated in salt. Some
postulate that the word soldier itself is derived from the word for salt. Even today, a hard-
working employee might be said to be “worth his salt” or might be commended for “soldiering
on.”
Questions

1. The author’s primary purpose is to


A) relate the history of salt.
B) describe the value of salt.
C) enumerate the myriad uses of salt.
D) discuss the areas where salt was traded.
E) explain the etymology of the word “salt.”

2. In line 21, engendering most closely means

A) causing.
B) encountering.
C) requiring.
D) restricting.
E) stopping.

3. The function of the passage’s final sentence is to


A) summarize the main uses of salt discussed in the passage.
B) explain the etymological roots of several city names.
C) repeat the fact that Roman soldiers were paid in salt.
D) illustrate the modern uses of salt.
E) emphasize the value of salt.
Answers and Explanations

1. The correct answer is B.


A) Incorrect. While the author discusses the history of salt in the first, third, and fourth
paragraphs, and focuses on the history of salt in lines 14-23, the history of salt is not the
main thrust of the passage. The author’s main purpose is to show the value of salt, and
she does so by enumerating its many uses, both now and throughout history, as she does
in lines 7-16, and in underlining its value by showing what a desired commodity it has
been through history. The author shows that salt has historically been a desired
commodity in lines 17-27. She underlines its value by showing that many languages
share the same root word for salt in lines 23-26 which suggests a long history of salt
trade. She then underlines her point by showing that even today’s language retains the
idea that salt is valuable, in the final sentence.
B) Correct. In every paragraph, the author discusses the value of salt. In line 5 of the first
paragraph, the phrase “… the king realized the value of salt and the depth of his
daughter’s love for him” mentions the value of salt. In lines 6 and 7 of the second
paragraph, the sentence “Saltiness is one of the basic tastes perceived by the tongue,
making it an esteemed and ubiquitous food flavoring” introduces the paragraph with the
idea of salt’s value by stating that salt is “esteemed,” which means valuable. Line 14
introduces the third paragraph with the phrase “Salt’s historical distinction lies not so
much in its taste…” showing the distinctiveness, or value of salt. And line 27 introduces
the fourth paragraph with the sentence “Throughout history, salt has been deemed a
precious commodity,” which also underlines the value of salt.
C) Incorrect. While the author does enumerate many uses for salt in lines 7-14 and 15-16,
she does so in order to illustrate the ways in which salt is valuable.
D) Incorrect. While the author does discuss the salt trade in lines 19–25, she does so in order
to illustrate the value of salt. For example, in lines 22-23, the author discusses the salt
trade with the phrase, “in the first millennium B.C., Celtic people exchanged salted meat
for wine and other luxury goods from ancient Greece and Rome” in order to show that
salt was traded for other valuable items.
E) Incorrect. While the author does discuss the etymology of various words associated with
the root words of salt in lines 25-34, she does so in order to illustrate the value of salt.
For example, in lines 27-29, the author writes “In fact, the word ‘salary’ is derived from
the Middle English salaire, from the Latin salarium, which means a payment made in salt
(sal) or for salt, from salarius which means ‘pertaining to salt”’ in order to underline the
value of salt.

2. The correct answer is A.

A) Correct. The salt trade caused wide-ranging trade associations throughout the
Mediterranean region
B) Incorrect. The phrase “engendering the establishment of wide-ranging trade associations
throughout the Mediterranean region” seems to rule out the possibility that engendering
would mean encountering. It is unlikely that the beginning of the salt trade would have
randomly encountered the beginning of wide–ranging trade in the area. It is more likely
that there would have been a cause-effect relationship between the two situations. The
following discussion of the shared root words for salt, “The wide expanse of the Celtic
salt trade is exemplified by the shared Celtic, Greek, and Egyptian root word for salt, hal,
which is iterated in the names of saltworks throughout the region: Halle and Schwäbisch
Hall in Germany, Halych in Ukraine, and Galicia in Spain” seems to seal the notion that
the salt trade caused increased regional trade.
C) Incorrect. While it is true that the beginning of the salt trade caused the increase in trade
in the region, it seems unlikely that the beginning of the salt trade required increased
trade. The section stating that “Egypt instituted exportation of salt fish to the Phoenicians,
who in turn traded Egyptian salt fish with their commercial partners throughout North
Africa, engendering the establishment of wide-ranging trade associations throughout the
Mediterranean region. Similarly, in the first millennium B.C., Celtic people exchanged
salted meat for wine and other luxury goods from ancient Greece and Rome” shows that
increased trade was a mutually advantageous development, but probably not a
requirement.
D) Incorrect. It is unlikely that the word engendering would mean restricting. The statement
that “Egypt instituted exportation of salt fish to the Phoenicians, who in turn traded
Egyptian salt fish with their commercial partners throughout North Africa, engendering
the establishment of wide-ranging trade associations throughout the Mediterranean
region. Similarly, in the first millennium B.C., Celtic people exchanged salted meat for
wine and other luxury goods from ancient Greece and Rome” indicates that trade was
increased, not restricted.
E) Incorrect. It is unlikely that the word engendering would mean stopping. The statement
that “Egypt instituted exportation of salt fish to the Phoenicians, who in turn traded
Egyptian salt fish with their commercial partners throughout North Africa, engendering
the establishment of wide-ranging trade associations throughout the Mediterranean
region. Similarly, in the first millennium B.C., Celtic people exchanged salted meat for
wine and other luxury goods from ancient Greece and Rome” indicates that trade was
increased, not stopped.

3. The correct answer is E.


A) Incorrect. While the passage does discuss the many uses of salt, including a section in the
final paragraph, “Some surmise that soldiers were remunerated in salt,” which postulates
one use of salt, the final sentences does not summarize these uses as much as it shows
that the value of salt is shown even in today’s idiom.
B) Incorrect. While the final paragraph does discuss etymology, as in lines 27-30 which
state, “In fact, the word ‘salary’ is derived from the Middle English salaire, from the
Latin salarium, which means a payment made in salt (sal) or for salt, from salarius which
means ‘pertaining to salt,”’ the section relating to city names is located in lines 25-26
which include the phrase “…root word for salt, hal, which is iterated in the names of
saltworks throughout the region: Halle and Schwäbisch Hall in Germany, Halych in
Ukraine, and Galicia in Spain,” and the final sentence is unrelated to city names; rather, it
shows that the value of salt is shown even in today’s language.
C) Incorrect. Lines 31-32 state, “Some surmise that soldiers were remunerated in salt. Some
postulate that the word soldier itself is derived from the word for salt,” however it is by
no means certain that soldiers were paid in salt or that the word solider itself is derived
from the word for salt. These are hypotheses, not facts. The final sentence shows that
even today’s language reflects the value of salt, but it does not say that Roman soldiers
were paid in salt.
D) Incorrect. The final sentence does not discuss the modern uses of salt. It discusses the
modern use of language regarding salt. The final sentence, “Even today, a hard-working
employee might be said to be ‘worth his salt’ or might be commended for ‘soldiering
on”’ does not mention the uses of salt.
E) Correct. The phrase “a hard-working employee might be said to be ‘worth his salt’”
means that the employee is as valuable as salt, pointing out the value of salt.
englishforeveryone.org Name________________
Date________________


Advanced Critical Reading – The Great Debates
The first of the Great Debates, between Senator John F. Kennedy of Massachusetts and
the incumbent Vice President Richard Nixon on September 26, 1960, centered around domestic
issues. The topic of the next debate, on October 7, was a clash over U.S. policy regarding two
small islands off the Chinese coast, and on October 13, this controversy continued. On October
5 21, the final debate, the candidates focused on American/Cuban relations.
Few of the 70 million viewers could have fathomed what this first–ever televised
presidential debate augured, not only for this specific series of debates, but more importantly for
the preeminent role the fledgling medium would play in the future of the political arena.
A pallid Nixon arrived at the Chicago CBS studios after a grueling day of campaigning.
10 The previous August a knee infection had sidelined him. He was still twenty pounds
underweight, and he perspired profusely in an ill–fitting shirt. Moreover, he declined makeup to
burnish his hospital pallor. The freshly–painted studio backdrop had dried to an ashen hue that
obscured his matching suit.
The Democratic contender by contrast exuded a robust glow after a month of
15 campaigning in California. He had spent his day rehearsing potential questions and relaxing. An
aide later admitted that he supplemented his natural glow with a smidge of makeup. He was fit,
trim, and confident.
Despite the remarkably similar agendas and arguments of the Republican and the
Democrat, TV viewers unequivocally believed Kennedy to be the victor – whereas people who
20 had followed the debates on the radio held the opposite opinion. The age of TV had arrived, and
the subsequent party shuffle proved the undeniable potency of television.

Questions
1. The author is mainly concerned about
A) the debating styles of John Kennedy and Richard Nixon during the 1960 Great Debates
B) the domestic issues which affected the result of 1960 Great Debates
C) the health of Richard Nixon at the time of the 1960 Great Debates
D) the number of television viewers who tuned in to the 1960 Great Debates
E) the effect of television on the results of the 1960 Great Debates

2. It can be inferred from the passage that


A) Kennedy was a better debater than Nixon
B) Nixon was the unequivocal winner of the 1960 debates
C) The Democrat beat the Republican in the 1960 election
D) Nixon was more prepared for the first debate than Kennedy
E) Kennedy and Nixon disagreed strongly on issues on the home front.

3. According to the passage, which of the following was true of Richard Nixon?
A) He had a five o’clock shadow during the first debate.
B) He wore a brown suit during the first debate.
C) He warned of the impending Cuban crisis.
D) He limped onstage for the first debate.
E) He lost his job after the election.
Answers and Explanations

1. The correct answer is E.


A) The debating styles of John Kennedy and Richard Nixon during the 1960 Great Debates
were similar, since TV viewers and radio listeners ended up with different opinions of
who won the debates.
B) The candidates had similar agendas and arguments, so domestic issues were not pivotal.
C) Richard Nixon was not sick at the time of the 1960 Great Debates. He was thin and pale,
but there is no mention that he was sick.
D) The number of television viewers who tuned in to the 1960 Great Debates was not the
author’s main concern.
E) Correct. The effect of television on the results of the 1960 Great Debates was the main
concern of the author

2. The correct answer is C.


A) Kennedy was not a better debater than Nixon: people who followed the debates on radio
thought Nixon had won the debates.
B) Nixon was not the unequivocal winner of the 1960 debates; people who watched the
debates on TV thought Kennedy was the winner.
C) Correct. The Democrat beat the Republican in the 1960 election; there was a party
shuffle. This is mentioned in the last line (21) of the passage. Since Nixon, the
Republican, was the incumbent, the shuffle resulted in Democrats taking office.
D) There is no mention of whether Nixon was more prepared for the first debate than
Kennedy. Kennedy rehearsed the day of the debate.
E) Kennedy and Nixon did not disagree strongly on issues on the home front. They had
similar agendas and arguments.

3. The correct answer is E.


A) He had a five o’clock shadow during the first debate. While this fact is widely known, it
is not mentioned in the passage.
B) He did not wear a brown suit during the first debate. His suit was obscured by the ashen–
colored – gray – paint.
C) Whether Nixon warned of the impending Cuban crisis was not mentioned in the passage.
D) Whether Nixon limped is not mentioned in the passage.
E) Correct. Nixon lost his job after the election. Since there was a party shuffle, the
incumbent lost his job. The incumbent Vice President was Richard Nixon.

S-ar putea să vă placă și